Решения - МГТУ ГА

advertisement
ФЕДЕРАЛЬНОЕ АГЕНТСТВО ВОЗДУШНОГО ТРАНСПОРТА
ФЕДЕРАЛЬНОЕ ГОСУДАРСТВЕННОЕ
ОБРАЗОВАТЕЛЬНОЕ УЧРЕЖДЕНИЕ
ВЫСШЕГО ПРОФЕССИОНАЛЬНОГО ОБРАЗОВАНИЯ
МОСКОВСКИЙ ГОСУДАРСТВЕННЫЙ ТЕХНИЧЕСКИЙ
УНИВЕРСИТЕТ ГРАЖДАНСКОЙ АВИАЦИИ
УДК 374.3
№ гос. регистрации
Инв. № 13_4
«УТВЕРЖДАЮ»
Проректор по НР и РСФ
профессор, доктор техн. наук
Е.Е. Нечаев
«___» декабря 2010г.
ОТЧЕТ
по договору № 75 от 02.11.2010 г.
на выполнению цикла мероприятий по социальному обслуживанию
населения в части предоставления образовательных услуг
Мероприятие 75.1
Развитие учебно-образовательного и воспитательного процесса в системе
«МГТУГА - средние школы САО г. Москвы», направленного на повышение
качества подготовки учащихся, с учетом образовательных стандартов,
условий сдачи ЕГЭ и участию в олимпиадном движении.
Раздел 75.1.3 (этап 4):
Разработка научно-методической базы проведения профильных олимпиад
Научный руководитель работ по Договору № 75/2010
проф., д.ю.н.
Б.П. Елисеев
Ответственный исполнитель работ по Договору № 75/2010
проф., д.ф.-м.н.
А.И. Козлов
Научный руководитель мероприятия 75.1.
проф., д.ф.н.
Б.П. Елисеев
Заместитель научного руководителя мероприятия 75.1.
проф., д.т.н.
А.И. Козлов
Руководитель научно-образовательного
коллектива раздела 75.1.3.
проф., д.т.н.
А.В. Самохин
Москва 2010г.
Коллектив исполнителей работ по разделу 75.1.3
Научно-образовательный коллектив
1. Руководитель коллектива проф., д.т.н.,
Самохин А.В.
2. доц., к.ф.-м. н.
3. проф., д.т.н.
4. доц., к.т.н.
5. проф., д.т.н.,
6. проф., д.т.н.
7. доц., к.т.н.
8. доц., к.ф.-м. н.
Дементьев Ю.И.
Камзолов С.К.
Колядов Д.В.
Машошин О.Ф.
Нечаев Е.Е.
Соловьева Т.Л.
Солодов В.В.
К выполнению организационной работы (сбор информационной
информации,
вычислительные
работы,
подбор
литературы,
оформительская работа, расчет заработной платы и т.д.) привлекались
также
1. Васильев А.В.
2. Рыбалкина А.Л.
1.3.4-252
75.1.3.4.
АПРОБАЦИЯ НАУЧНО-ОБРАЗОВАТЕЛЬНЫХ И НАУЧНО –
ИНФОРМАЦИОННЫХ МАТЕРИАЛОВ ПО ПРОВЕДЕНИЮ
ПРОФИЛЬНЫХ ОЛИМПИАД В ШКОЛАХ САО Г. МОСКВЫ
ПОЛОЖЕНИЕ
ОБ ОТРАСЛЕВОЙ ОЛИМПИАДЕ
МОСКОВСКОГО ГОСУДАРСТВЕННОГО ТЕХНИЧЕСКОГО
УНИВЕРСИТЕТА ГРАЖДАНСКОЙ АВИАЦИИ
В 2010-2011 УЧЕБНОМ ГОДУ
1. Общие положения
1.1. Настоящее Положение об отраслевой олимпиаде Московского
Государственного Технического Университета Гражданской Авиации
(далее – Положение) определяет правила организации и проведения
отраслевой олимпиады Московского Государственного Технического
Университета
Гражданской
Авиации
(далее
–
Олимпиада),
ее
организационно-методическое обеспечение, правила участия в Олимпиаде
и определения победителей и призеров, права победителей и призеров
Олимпиады.
1.2. Настоящее положение составлено на основании Порядка
проведения
олимпиад
школьников,
утвержденного
приказом
Министерства образования и науки Российской Федерации от 22 октября
2007 г. N 285 (зарегистрирован Министерством юстиции Российской
Федерации 16 ноября 2007 г., регистрационный N 10496) в редакции
приказов Министерства образования и науки Российской Федерации от 4
сентября 2008 г. N 255 (зарегистрирован Министерством юстиции
Российской Федерации 1 октября 2008 г., регистрационный N 12381), от 20
марта 2009 г. N 92 (зарегистрирован Министерством юстиции Российской
Федерации 27 апреля 2009 г., регистрационный N 13837), от 6 октября
2009 г. N 371 (зарегистрирован Министерством юстиции Российской
1.3.4-253
Федерации 24 ноября 2009 г., регистрационный N 15301) и от 11 октября
2010 г. N 1006 (зарегистрирован Министерством юстиции Российской
Федерации 22 октября 2010 г., регистрационный N 18789).
1.3. Основными целями Олимпиады являются выявление и развитие
у
обучающихся
творческих
способностей
и
интереса
к
научно-
исследовательской деятельности, создание необходимых условий для
поддержки одарённых детей, распространение и популяризация научных
знаний среди молодежи.
1.4.
Олимпиада
проводится
Московским
Государственным
Техническим Университетом Гражданской Авиации ежегодно в два этапа
в очной форме, в Москве и в филиалах Московского Государственного
Технического Университета Гражданкой Авиации.
1.5. Положение об Олимпиаде утверждается ректором Московского
Государственного Технического Университета Гражданкой Авиации.
1.6. Олимпиада проводится по следующим предметам: математика,
физика. Олимпиада проводится по заданиям, составленным на основе
примерных основных общеобразовательных программ основного общего и
среднего (полного) общего образования. Рабочим языком проведения
Олимпиады является русский язык. Результаты участников Олимпиады
оцениваются по 30-балльной шкале.
1.7. В Олимпиаде на добровольной основе принимают участие
обучающиеся
образовательных
учреждений,
осваивающие
общеобразовательные программы среднего (полного) общего образования.
Правила участия в каждом этапе олимпиады определяются настоящим
Положением об Олимпиаде.
1.3.4-254
2. Порядок организации, сроки проведения и
правила участия в Олимпиаде
2.1. Олимпиада проводится по утвержденному председателем
оргкомитета расписанию с 1 декабря по 31 марта и состоит из двух этапов:
- отборочный этап, который проводится в очной форме в декабре;
- заключительный этап, который проводится в очной форме в
период с 1 февраля по 31 марта.
2.2. К участию в отборочном этапе допускается любой учащийся
образовательных
учреждений,
осваивающий
общеобразовательные
программы среднего (полного) общего образования. К участию в
заключительном этапе допускаются победители и призеры отборочного
этапа Олимпиады. В заключительном этапе Олимпиады могут участвовать
победители и призеры предшествующего года аналогичной Олимпиады в
случае, если они продолжают освоение общеобразовательных программ
среднего (полного) общего образования. Подведение итогов Олимпиады
проводится по результатам личного (индивидуального) зачета.
2.3. Консультации по предметам Олимпиады не проводятся.
2.4. Информация об Олимпиаде, порядке участия в ней и учета
результатов
участников
Олимпиады
публикуется
на
web-сайте
Московского Государственного Технического Университета Гражданской
Авиации не позднее, чем за неделю до начала проведения Олимпиады.
2.5. Информация о победителях и призерах этапов Олимпиады
является открытой и публикуется в сети Интернет на web-сайте
Московского Государственного Технического Университета Гражданской
Авиации
не
позднее,
чем
через
две
недели
после
проведения
соответствующего этапа Олимпиады.
2.6.
Представление
отчетной
документации,
размещение
информации о победителях и призерах Олимпиады на web-сайте
Московского Государственного Технического Университета Гражданской
1.3.4-255
Авиации осуществляются в срок до 10 апреля; вручение дипломов
победителям и призерам Олимпиады осуществляется в срок до 15 мая.
2.7. Для организационно-методического обеспечения проведения
Олимпиады
приказом
ректора
Московского
Государственного
Технического Университета Гражданской Авиации создается оргкомитет
Олимпиады на срок проведения Олимпиады.
3. Функции оргкомитета, предметных методических комиссий и
предметных жюри
3.1. Состав оргкомитета Олимпиады утверждает ректор Московского
Государственного Технического Университета Гражданской Авиации.
3.2. Оргкомитет Олимпиады:
- представляет в Совет олимпиад документы, необходимые для
включения Олимпиады в перечень олимпиад;
- формирует составы предметных методических комиссий и
предметных жюри Олимпиады, одновременное членство лиц в предметной
методической комиссии и в предметном жюри Олимпиады не допускается;
- обеспечивает
Оргкомитет
непосредственное
Олимпиады
может
проведение
привлекать
к
Олимпиады.
непосредственному
проведению Олимпиады членов предметных методических комиссий,
членов предметных жюри, профессорско-преподавательский состав и
студентов;
- рассматривает
совместно
с
предметными
методическими
комиссиями и предметными жюри Олимпиады апелляции участников
Олимпиады и принимает окончательные решения по результатам их
рассмотрения;
- утверждает список победителей и призеров Олимпиады;
- награждает победителей и призеров Олимпиады;
1.3.4-256
- представляет в Совет олимпиад ежегодный отчет по итогам
прошедшей Олимпиады;
- осуществляет иные функции в соответствии с Положением об
Олимпиаде.
3.3. Предметные методические комиссии:
- разрабатывают и размножают материалы олимпиадных заданий
для этапов Олимпиады;
- разрабатывают критерии и методики оценки выполненных
заданий всех этапов Олимпиады;
- представляют
в
оргкомитет
Олимпиады
предложения
по
вопросам, связанным с совершенствованием организации проведения
Олимпиады;
- рассматривают
совместно
с
оргкомитетом
Олимпиады
и
предметными жюри Олимпиады апелляции участников Олимпиады;
- публикуют решения олимпиадных заданий;
- осуществляют иные функции в соответствии с Положением об
Олимпиаде.
3.4. Предметные жюри:
- проверяют и оценивают результаты выполнения олимпиадных
заданий и других видов испытаний участниками Олимпиады;
- определяют кандидатуры победителей и призеров Олимпиады;
- проводят показ работ для участников Олимпиады;
- рассматривают
предметными
совместно
методическими
с
оргкомитетом
комиссиями
Олимпиады
Олимпиады
и
апелляции
участников Олимпиады;
- осуществляют иные функции в соответствии с Положением об
Олимпиаде.
1.3.4-257
4. Порядок определения победителей и призеров Олимпиады
4.1. Победители и призеры этапов Олимпиады определяются на
основании результатов участников соответствующих этапов Олимпиады.
Количество победителей каждого этапа Олимпиады по
4.2.
соответствующему
общеобразовательному
предмету,
по
которому
проводится олимпиада, не должно превышать 10 процентов от общего
числа
участников
соответствующего
этапа
Олимпиады
по
соответствующему общеобразовательному предмету. Общее количество
победителей
и
призеров
каждого
этапа
Олимпиады
по
общеобразовательному предмету, по которому проводится олимпиада, не
должно
превышать
соответствующего
35
процентов
этапа
от
Олимпиады
общего
по
числа
участников
соответствующему
общеобразовательному предмету.
4.3.
результатам
Победители
и
призеры
заключительного
этапа
Олимпиады
определяются
Олимпиады.
по
Победителями
Олимпиады считаются участники Олимпиады, награжденные дипломами 1
степени. Призерами Олимпиады считаются участники Олимпиады,
награжденные дипломами 2 и 3 степени.
4.4. Дипломы победителей и призеров Олимпиады подписываются
председателем оргкомитета Олимпиады.
5. Льготы победителям и призерам Олимпиады
5.1. При равенстве общего числа баллов преимущественным правом
поступления в Московский Государственный Технический Университет
Гражданской Авиации пользуются победители и призеры Олимпиады;
5.2. Победителям и призерам Олимпиады могут быть учтены
результаты Олимпиады при поступлении в Московский Государственный
Технический Университет Гражданской Авиации по соответствующему
предмету только в текущем году набора.
1.3.4-258
6. Финансовое обеспечение Олимпиады
6.1. Финансовое обеспечение Олимпиады осуществляется за счет
средств Московского Государственного Технического Университета
Гражданской Авиации.
6.2. Взимание платы (в какой-либо форме) за участие в Олимпиаде
не допускается.
7. Порядок проведения Олимпиады школьников
К
участию
образовательных
в
Олимпиаде
учреждений
допускаются
Российской
любые
Федерации,
учащиеся
осваивающие
общеобразовательные программы среднего (полного) общего образования.
Члены методической комиссии рассаживают участников Олимпиады
в заранее подготовленные аудитории.
Участникам Олимпиады раздаются титульный лист, беловик и
черновик. С помощью представителя оргкомитета участники Олимпиады
заполняют
личные
соответствующей
данные
графе
участника
ведомости
на
титульном
участник
ставит
листе.
В
подпись,
удостоверяющую правильность внесённых данных. Особое внимание
следует обратить на разборчивую запись всех личных данных и на запрет
подписывать беловики и черновики.
Перед получением задания участников знакомят с основными
правилами проведения Олимпиады:
1. При выполнении задания Олимпиады участники используют
ручки с чернилами или пастой черного, синего или фиолетового цвета.
2. Во время выполнения задания не допускается использование
участниками учебников, справочников, таблиц функций или формул,
шпаргалок.
1.3.4-259
3. Пользование калькуляторами и иными электронными средствами
запрещено. Средства мобильной связи должны быть отключены.
4. Любые разговоры между участниками должны быть исключены,
вопросы могут задаваться лишь присутствующим членам методических
комиссий.
5. Если участнику Олимпиады понадобится дополнительный беловик
или черновик, он ему выдается. На титульном листе представитель
оргкомитета ставит об этом соответствующую пометку.
Нарушители правил удаляются из аудитории, а их работы
аннулируются, о чем составляется специальный акт представителями
оргкомитета и методической комиссии.
После заполнения личных данных участников олимпиады им
раздаются олимпиадные задания обратной стороной вверх. Затем все
одновременно переворачивают задания, и с этого момента отсчитывается
написание Олимпиады.
На выполнение задания участникам отводится 180 минут.
По окончании выполнения задания, участники сдают работы
представителю оргкомитета и после этого покидают аудиторию. В
обязательном порядке участники Олимпиады должны сдать все выданные
им листы (включая дополнительные).
8. Порядок проведения апелляции
по результатам проверки заданий олимпиады
Московского технического университета гражданской авиации
8.1.
Основные положения
Настоящий порядок проведения апелляции по результатам
проверки
заданий
(далее
Порядок)
олимпиады
технического университета гражданской авиации
Московского
(далее Олимпиады)
разработан в соответствии с:
1.3.4-260
Федеральным законом «Об образовании» от 10.07.1992 № 3266-1;
Законом г. Москвы «О развитии образования в городе Москве» от
20.06.2001 № 25
Положением
утвержденным
о
приказом
Всероссийской
Министерства
олимпиаде
школьников,
образования
Российской
Федерации от 30.10.2003 № 4072.
Апелляцию по результатам проверки заданий
Олимпиады (далее
апелляция) может подать любой участник Олимпиады.
Процедура подачи и рассмотрения апелляции доводится до
сведения участников Олимпиады за неделю до начала проведения
Олимпиады путем размещения Порядка на сайте МГТУГА.
8.2.. Срок и порядок проведения апелляции
Апелляция проводится в день показа работ, который назначается
председателем оргкомитета Олимпиады и доводится до сведения
участников Олимпиады путем размещения сообщения на сайте МГТУГА.
В день показа работ любой участник Олимпиады имеет право в
присутствии членов жюри ознакомиться с результатами проверки
своей работы.
В
ходе
ознакомления
с
работой
участнику
Олимпиады
категорически запрещается вносить какие-либо изменения в текст
работы. В случае попытки нарушения этого положения участник
Олимпиады немедленно лишается права на апелляцию, о чем члены
жюри письменно сообщают председателю Оргкомитета Олимпиады.
В случае несогласия с выставленными баллами в день показа работ
участник Олимпиады, не выходя из аудитории, в которой проходит
показ работ, подает письменное апелляционное заявление на имя
председателя
Оргкомитета,
который
одновременно
является
председателем апелляционной комиссии.
В апелляционном заявлении должны быть мотивированы причины,
1.3.4-261
по которым участник Олимпиады не согласен с выставленными баллами.
Апелляционные заявления от иных лиц, в том числе родственников
участника Олимпиады, не принимаются и не рассматриваются.
Апелляция для участников Олимпиады, не подавших заявление в
установленный срок, не назначается и не
проводится,
претензии не
рассматриваются.
Участник
Олимпиады
имеет
право
присутствовать
при
рассмотрении апелляции. При этом он должен иметь при себе
документ, удостоверяющий личность.
С несовершеннолетним участником олимпиады (до 18 лет) на
апелляции имеет право присутствовать один из родителей или
законных представителей, (кроме несовершеннолетних, признанных в
соответствии с законом полностью дееспособными до достижения
совершеннолетия).
В ходе рассмотрения апелляции проверяется только правильность
выставленных
баллов.
Дополнительный
опрос
при
рассмотрении
апелляции участника Олимпиады не допускается.
По результатам рассмотрения апелляции апелляционная комиссия
может принять одно из решений:
- об отклонении апелляции и подтверждении выставленных баллов;
- об удовлетворении апелляции и выставлении новых баллов.
8.3. Порядок организации и работы апелляционной комиссии
Для проведения апелляции создается апелляционная комиссия.
Персональный состав апелляционной комиссии
определяется
председателем Оргкомитета из состава предметной методической
комиссии и оргкомитета (не менее трех, но не более семи человек) в
день проведения апелляции.
Делопроизводство
апелляционной
комиссии ведет секретарь,
назначаемый председателем апелляционной комиссии из числа ее членов.
1.3.4-262
Апелляционная комиссия выполняет следующие функции:
- принимает и рассматривает апелляции участников олимпиады;
- выносит решение по результатам рассмотрения апелляции;
Апелляционная
комиссия
осуществляет
свою
работу
принимаются
простым
непосредственно в день показа работ.
Решения
апелляционной
комиссии
большинством голосов ее членов и оформляются протоколами,
которые подписываются председателем и всеми членами комиссии.
Решения апелляционной комиссии являются окончательными и
пересмотру не подлежат.
Протоколы проведения апелляции передаются
оргкомитета и председателю жюри для внесения
председателю
соответствующих
изменений в итоговую документацию.
9. Сведения о наличии необходимых организационных, методических,
кадровых, материально-хозяйственных и финансовых ресурсах.
9.1. В Университете работает постоянно действующий оргкомитет
Олимпиады. Его персональный состав ежегодно утверждается приказом
ректора.
9.2.
Методическое
обеспечение
Олимпиады
находится
в
компетенции предметных методических комиссий. Задания проходят
утверждение в городских методических комиссиях по математике, физике
и русскому языку.
9.3.
В
состав
предметных
методических
комиссий
входят
заведующие кафедрами высшей математики и физики и преподаватели
этих кафедр, имеющие высокую методическую квалификацию.
9.4.
имеющие
Большую
часть
многолетний
оргкомитета
стаж
работы
составляют
в
преподаватели,
олимпиадном
движении.
Председатель оргкомитета, профессор, доктор наук А.В. Самохин, трижды
1.3.4-263
Соросовский доцент по математике, участвует в организации олимпиад
разного уровня более 40 лет.
9.5. Университет Гражданской Авиации выделяет для проведения
Олимпиад, разбора олимпиадных заданий, показа работ и апелляций
несколько 200-местных аудиторий, обеспечивает хранение работ и выдачу
дипломов.
9.6. Все расходы на проведение Олимпиады, согласно «Положению
об Олимпиаде школьников», принимает на себя Университет. Ежегодно
университетом выделяются денежные средства в размере ста тысяч рублей
на информационное и организационное обеспечение олимпиады.
1.3.4-264
Разработка рекламных материалов для своевременного
оповещения о проведении олимпиады
в средствах массовой информации, Интернете.
Подготовка материалов к изданию рекламных буклетов
Информационное обеспечение олимпиады школьников
Информационное обеспечение олимпиады школьников производится
по следующим направлениям:
1. Размещение информации об олимпиаде и сроках её проведения на
сайте
олимпиад
МГТУ
ГА
http://olimp.mstuca.ru
и
на
сайте
Государственной службы гражданской авиации
2. Размещение правил проведения олимпиады и нормативных
документов на сайте олимпиад МГТУ ГА http://olimp.mstuca.ru.
3. Сообщение о проведении олимпиады на Дне открытых дверей
университета.
4. Размещение информации об олимпиаде в сети Интернет.
5. Печать объявлений в отраслевых и в окружных газетах.
6. Размещение плакатов в школах Северного административного
округа г. Москвы.
7. Размещение плакатов в аэропортах и в авиаремонтных
предприятиях.
8. Размещение рекламных буклетов среди школьников и учителей в
школах Северного административного округа г. Москвы.
9. Размещение вариантов и результатов олимпиады на сайте
олимпиад МГТУ ГА http://olimp.mstuca.ru.
10. Объявление дат разбора олимпиадных заданий, просмотра работ
и награждения победителей и призёров через сайт олимпиад МГТУ ГА
http://olimp.mstuca.ru.
1.3.4-265
Подготовка материалов к изданию рекламных буклетов
В рекламных буклетах будет приведена исчерпывающая информация
по олимпиаде. Будет указано, по каким предметам проводится олимпиада,
даты и время проведения олимпиад, адрес и проезд до места её
проведения, а также телефоны и электронные адреса для вопросов
школьников и их родителей. В буклете дополнительно будут приведены
варианты олимпиадных заданий за прошлые годы, краткая история
проведения олимпиад в МГТУ ГА, а также краткие сведения об МГТУГА.
Сведения об истории проведения
Отраслевой Олимпиады школьников
Наш университет образован в 1971 году. Первоначально он назывался
Московский институт инженеров гражданской авиации (МИИГА). Спустя
несколько лет, в 1975 году, группа преподавателей кафедр физики и
математики
предложила
провести
на базе института
Отраслевую
Олимпиаду школьников по физике и математике для учащихся 10-х
классов. Инициативной группе хотелось, прежде всего, оценить уровень
подготовки школьных выпускников, их умение логически мыслить и
использовать свои знания для решения нетрадиционных задач.
Ученый Совет института эту идею одобрил и сразу же в своем решении
определил основные принципы проведения Олимпиады: общедоступно,
добровольно,
доступно. При этом указанные в решении принципы
распространялись как на учащихся, принимающих участие в Олимпиаде,
так и на сотрудников и студентов нашего института, организующих
проведение Олимпиады.
С этого момента ежегодно, традиционно, в мае месяце проводится
Отраслевая Олимпиада МГТУГА.
Прошло уже более тридцати лет. В нашей стране стала развиваться
рыночная экономика. Время от времени в институте высказывались
1.3.4-266
различные
идеи
об
изменении
характера
Олимпиады.
Поступали
предложения проводить ее несколько раз в году, взимать плату за участие
и оплачивать из этих денег труд организаторов, приравнять ее к
вступительным
экзаменам,
внеся
соответствующие
изменения
в
олимпиадные задания, проводить Олимпиаду отдельно для выпускников
подготовительных курсов и подшефных школ с целью их льготного
зачисления в институт и т.п. Но Ученый совет нашего, теперь уже
университета, ни разу не отошел от определенных раз и навсегда
принципов и отклонил абсолютно все предложения такого рода.
Твердая позиция нашего Ученого совета, оргкомитета Олимпиады
принесла свои плоды. Ежегодно к нам на Олимпиаду приходит от ста до
двухсот человек. В большинстве своем это жители нашего округа, Москвы
и Подмосковья. Большой интерес вызывает Олимпиада у школьников
Подмосковья, проживающих в районах аэропортов и авиаремонтных
предприятий.
В последние годы стала прослеживаться определенная
тенденция – среди школьников становится престижным попробовать свои
силы, блеснуть смекалкой и получить диплом Олимпиады. И идут они
туда иногда даже не имея планов поступать в наш университет. Уже
возникло своеобразное соревнование среди учителей физики и математики
нашего округа на предмет того, чьи ученики получат больше дипломов на
Олимпиаде.
На сегодняшний день основными целями и задачами нашей Олимпиады
являются повышение интереса школьников к изучению точных наук,
выявление способных подростков и содействие им в профессиональной
ориентации и получении высшего образования, определение сильных и
слабых сторон подготовки школьников с целью повышения уровня
преподавания точных дисциплин на младших курсах нашего университета.
Большую часть оргкомитета Олимпиады составляют преподаватели с
многолетним стажем работы в олимпиадном движении. Председатель
1.3.4-267
оргкомитета, профессор, доктор наук А.В. Самохин, трижды Соросовский
доцент по математике, участвует в организации олимпиад разного уровня
более сорока лет.
Учитывая определенную социальную напряженность в нашем округе,
распространение
обеспокоенность
негативных
тенденций
в
профессорско-преподавательского
кругах
молодежи,
состава
уровнем
знаний и интересов выпускников школ, малое количество мест для
проведения бесплатного досуга старшеклассников, Ученый совет нашего
института и оргкомитет Олимпиады считают целесообразным начиная со
следующего года привлечь к Олимпиаде не только выпускников школ, но и
учащихся 10-х классов. Наш университет готов предоставить возможность
победителям и призерам Олимпиады среди учащихся 10-х классов (до 45%
от числа участников) пользоваться библиотечными фондами, наравне со
студентами разрабатывать и испытывать новые дельталеты в Студенческом
конструкторском бюро, осваивать компьютерные программы в Центре
новых
информационных
технологий,
посещать
секции
волейбола,
баскетбола, футбола, настольного тенниса, легкой атлетики, атлетической
гимнастики и ритмической гимнастики, заниматься в театральной и
танцевальной (современный и народный танец) студии, студии эстрадного и
академического вокала, в вокально-инструментальных ансамблях и коллективе
КВН, студии художественного слова.
Сведения об истории проведения окружного этапа
Московской региональной Олимпиады школьников
В 2008 году окружной этап Московской региональной Олимпиады
школьников в Московском государственном техническом университете
гражданской авиации проводился впервые.
Наш
школьников
университет
по
включился
решению
Ученого
во
Всероссийскую
совета
олимпиаду
университета,
который
поддерживает все инициативы, направленные на выявление и развитие у
1.3.4-268
учащихся творческих способностей и повышение интереса школьников к
изучению точных наук.
В организации этой Олимпиады нам в значительной степени помог
опыт
проведения
отраслевой
Олимпиады
школьников,
которая
традиционно проходит в нашем университете в течение более тридцати
лет.
Для
проведения
Олимпиады
на
добровольной
основе
были
сформированы оргкомитет, методические комиссии и жюри.
Большую часть оргкомитета составили преподаватели с многолетним
стажем работы в олимпиадном движении. Председатель оргкомитета,
профессор, доктор наук А.В. Самохин, трижды Соросовский доцент по
математике, участвует в организации олимпиад разного уровня более
сорока лет.
В составе предметных методических комиссий Олимпиады работали
профессора
университета
и
представители
педагогической
общественности Северного административного округа, имеющие высокую
методическую квалификацию.
В жюри Олимпиады вошли, в основном, молодые преподаватели,
изъявившие желание оценить сильные и слабые стороны подготовки
школьников с целью повышения уровня преподавания точных дисциплин
на младших курсах нашего университета.
Информационное обеспечение Олимпиады было поручено молодым
преподавателям и студентам университета.
Олимпиада, в целом, в нашем университете прошла хорошо.
Конфликтных
ситуаций
и
апелляционных
заявлений
не
было.
Профессорско-преподавательский состав специальных кафедр, который не
принимал непосредственного участия в проведении Олимпиады, отнесся с
большим интересом к ее результатам, и еще долгое время после окончания
Олимпиады обсуждал это мероприятие.
1.3.4-269
Ученый совет университета положительно оценивает результаты
проведения Олимпиады и считает целесообразным продолжить участие
университета во Всероссийской Олимпиаде школьников. Аналогичные
пожелания высказывает педагогическая общественность САО, с которой у
университета имеются давно налаженные связи.
Краткие сведения об МГТУГА
Федеральное государственное образовательное учреждение высшего
профессионального образования «Московский государственный технический
университет гражданской авиации» (МГТУГА) готовит специалистов с
высшим образованием
для гражданской авиации (ГА) начиная
с
1971г.Учебную и научно-исследовательскую работу ведут кафедры и научноисследовательские подразделения университета, возглавляемые опытными
учеными-педагогами, среди которых - академики, заслуженные деятели науки
и техники, высшего образования, транспорта РФ. Более 25% преподавателей доктора наук, профессора. За 37 лет подготовлено около 18500 инженеров,
более 300 магистров и бакалавров техники и технологии. В системе
повышения квалификации прошли обучение более 4000 руководителей
центрального аппарата управления отраслью, эксплуатационных предприятий
и организаций, преподавателей учебных заведений и ведущих специалистов
инженерно-авиационной службы, коммерческих подразделений гражданской
авиации.
В настоящее время в структуре университета 6 факультетов, 4
крупных научно-исследовательских лабораторий, центр переподготовки и
повышения квалификации кадров воздушного транспорта Российской
Федерации. Университет имеет филиалы в г.г. Ростов-на-Дону, Иркутск,
Кирсанов, Троицк, Егорьевск и Рыльск.
В МГТУ ГА имеется аспирантура, докторантура. Университет
является одним из крупных научно-исследовательских центров отрасли. Здесь
1.3.4-270
выполняются
фундаментальные
научные
исследования
и
опытно-
конструкторские работы по проблемам безопасности полетов, технического
обслуживания и ремонта авиационной техники, радиолокации и управления
воздушным движением, коммерческой эксплуатации воздушного транспорта,
менеджмента
авиатранспортного
производства,
методологии
высшего
образования.
Учебные и научные лаборатории оснащены новейшей аппаратурой,
современными
действующими
компьютерами,
образцами
стендами,
авиационного
макетами,
и
тренажерами,
радиоэлектронного
оборудования, техническими средствами обучения и контроля знаний.
Единая университетская локальная вычислительная сеть (более 500
ЭВМ) включает компьютерные классы для групповых и индивидуальных
занятий
студентов,
автоматизированные
информационную
рабочие
места
систему
управленческих
библиотеки,
подразделений,
программно-аппаратный комплекс с выходами во внешние компьютерные
сети. В университете созданы и успешно работают студенческие научные
подразделения, в которых студенты совершенствуют теоретические знания и
приобретают практические навыки работы, они занимаются проектированием
и постройкой легких летательных аппаратов, разработкой и модернизацией
авиационного
оборудования
и
радиоэлектронных
систем. Регулярно
проводятся олимпиады, научно-технические конференции студентов. Лучшие
работы
представляются
на
Российские
и
международные конкурсы
студенческих научных работ. Студенты университета нередко становятся
лауреатами таких конкурсов. На учебном аэродроме учебного авиационнотехнического центра университета в районе аэропорта Шереметьево имеются
различные типы самолетов и вертолетов гражданской авиации России (Ил-86,
Ил-76, Ил-62М, Ту-154М, Як-40, Ан-24, Ми-2, Ми-8), на которых проводятся
большинство практик. Радиополигон УБ ЭРТОС оснащен всеми видами
1.3.4-271
наземных радиолокационных комплексов и радиотехнических посадочных
средств.
Библиотека университета (более 1 млн. книг) обеспечена учебной
литературой по профилю всех факультетов, актуальным проблемам науки и
техники, общественным наукам. Она является одной из лучших библиотек
России по авиационной технике. К услугам читателей - комфортабельные
читальные
залы
с
электронными
каталогами
и
множительной
техникой. Иногородние студенты обеспечиваются местами в общежитии,
расположенном в парковой зоне, недалеко от университета и метро.
Вместительный киноконцертный зал с репетиционными помещениями и
многофункциональный
обеспечивают
все
оздоровительной
спортивные
спортивный
условия
работы.
секции.
В
для
комплекс,
культурно-массовой
университете
Спортивный
включающий
работают
комплекс
и
стадион,
спортивно-
многочисленные
включает
стадион
с
легкоатлетическим ядром и футбольным полем, специализированные залы для
спортивных игр, теннисные корты, лыжную базу и функциональнооздоровительный центр. Сборные команды и спортсмены университета
неоднократно становились чемпионами и призерами России и Москвы среди
учебных заведений. На время обучения в университете студентам дневного
отделения предоставляется отсрочка от призыва на военную службу. Военной
кафедры в университете нет. Университет осуществляет подготовку
специалистов по дневной и заочной формам обучения.
Начиная с 1992/1993 учебного года, в МГТУ ГА введена новая
многоуровневая система высшего образования, соответствующая европейским
стандартам. Данная система предусматривает получение высшего базового
(бакалавр) и высшего специального образования (специалист, магистр). В
1.3.4-272
МГТУ ГА осуществляется подготовка по следующим направлениям и
специальностям:
1. Направление "Эксплуатация и испытания авиационной и космической
техники" (шифр 160900). Квалификация выпускника: "Бакалавр техники и
технологии", срок обучения 4 года; "Магистр техники и технологии", срок
обучения - 6 лет.
2.1. Специальность "Техническая эксплуатация летательных аппаратов и
двигателей" (шифр 160901). Квалификация выпускника: инженер, срок
обучения - 5 лет 6 месяцев.
2.2.
Специальность
"Техническая
эксплуатация
авиационных
электросистем и пилотажно-навигационных комплексов" (шифр 160903).
Квалификация выпускника: инженер, срок обучения - 5,5 лет.
2.3.
Специальность
«Техническая
эксплуатация
транспортного
радиооборудования» (шифр 160905). Квалификация выпускника: инженер,
срок обучения - 5,5 лет.
3. Направление "Информатика и вычислительная техника" (шифр
230100). Специальность "Вычислительные машины, комплексы, системы и
сети" (шифр 230101). Квалификация выпускника: инженер, срок обучения - 5
лет.
4. Направление «Безопасность жизнедеятельности» (шифр 280100).
Специальность «Безопасность технологических процессов и производств»
(шифр 280102). Квалификация выпускника: инженер, срок обучения – 5 лет.
5.
Направление
"Прикладная
математика"
(шифр
230400).
Специальность "Прикладная математика" ( шифр 230401). Квалификация
выпускника: инженер-математик, срок обучения - 5 лет.
1.3.4-273
6.
Специальность
"Менеджмент
организации"
(шифр
080507).
Квалификация выпускника: менеджер, срок обучения - 5 лет.
7. Специальность
«Связи с общественностью» (шифр 030602).
Квалификация выпускника: специалист по связям с общественностью, срок
обучения – 5 лет.
8. Специальность
«Информационная
безопасность
телекоммуникационных систем» (шифр 090106). Квалификация выпускника:
специалист по защите информации, срок обучения – 5,5 лет.
9. Специальность «Организация перевозок и управление на транспорте»
(шифр 190701). Квалификация выпускника: инженер по организации
управления на транспорте, срок обучения -5 лет.
10. Специальность «Аэронавигационное обслуживание и использование
воздушного пространства - "УВД" (160505)», квалификация выпускника инженер, срок обучения - 5 лет.
Важнейшей особенностью высшего базового образования является
обучение в течение 4 лет по направлению, охватывающему более широкую
область знаний, чем одна специальность. Это направление является базой для
ряда органически связанных между собой специальностей. Студент МГТУ ГА,
получивший высшее базовое образование, может продолжить обучение в
МГТУ ГА на следующем уровне образования (высшее специальное
образование). Высшее специальное образование, в основе которого лежит
базовое высшее образование, завершается либо получением диплома
инженера по конкретной специальности, либо диплома магистра.
Программа подготовки дипломированных инженеров включает в себя
защиту дипломного проекта по специальности. Программа подготовки
1.3.4-274
магистров имеет научно-исследовательскую направленность и реализуется
параллельно подготовке инженеров. По этой программе обучаются наиболее
способные к научной работе студенты, причем обучение ведется по
индивидуальным планам под непосредственным руководством ведущих
профессоров МГТУ ГА. Университет готовит высококвалифицированных
специалистов (инженеров и магистров) преимущественно на основе базового
высшего образования с глубокими фундаментальными и прикладными
знаниями в соответствующих областях деятельности. Подготовка с неполным
высшим образованием и присвоением квалификации техника в МГТУ ГА не
производится.
ИНФОРМАЦИЯ
При университете работают подготовительные курсы:
Телефон: 459-07-29.
Адрес университета: 125493, г. Москва, Кронштадтский бульвар, 20
(УЛК). Телефон приемной комиссии: 458-75-47, с 20 июня по 10 августа
459-07-40.
1.3.4-275
Разработка критериев оценивания олимпиадных заданий и
критериев определения победителей и призеров олимпиады
Критерии оценивания заданий по математике
Задания математических олимпиад являются творческими,
допускают несколько различных вариантов решений. Кроме того,
необходимо оценивать частичные продвижения в задачах (например,
разбор важного случая, доказательство леммы, нахождение примера и
т.п.). Наконец, возможны логические и арифметические ошибки в
решениях. Окончательные баллы по задаче должны учитывать все
вышеперечисленное.
Общие положения по проверке
Предлагается сначала проверять в традиционных "плюс–
минусах", после чего переводить оценку в баллы. Все задачи оцениваются
в 7 баллов.
+ (плюс) (7)
Задача решена верно.
+. (плюс с точкой) (6)
Задача решена, но в записи решения
имеются легко устранимые пропуски.
+– (плюс - минус) (5)
Задача решена, однако пропуски в
записи решения существенны, например не пояснены (не доказаны)
существенные утверждения.
+/2 (плюс пополам) (4)
Такая оценка обычно ставится, когда
трудно принять решение: решена задача или нет.
–+ (минус - плюс) (2–3)
Задача не решена, но многие важные
идеи решения имеются, правилен подход к решению задачи.
–. (минус с точкой) (1)
Решение неверно, но имеются
разумные соображения, которые могут быть доведены до правильного
решения.
1.3.4-276
– (минус) (0)
Задача решена неверно.
О
Отсутствует решение.
(0)
В соответствии с регламентом проведения математических олимпиад
школьников каждая задача оценивается из 7 баллов.
Соответствие правильности решения и выставляемых баллов
приведено в таблице.
Баллы
7
Правильность (ошибочность) решения
Полное верное решение
0
Верное решение. Имеются небольшие недочеты, в целом не
влияющие на решение.
Решение в целом верное. Однако решение содержит
существенные ошибки либо пропущены случаи, не
влияющие на логику рассуждений.
Верно рассмотрен один из двух (более сложный)
существенных случаев, или в задаче типа «оценка + пример»
верно получена оценка.
Доказаны вспомогательные утверждения, помогающие в
решении задачи.
Рассмотрены отдельные важные случаи при отсутствии
решения (или при ошибочном решении).
Решение неверное, продвижения отсутствуют.
0
Решение отсутствует.
6-7
5-6
4
2-3
0-1
Важно отметить, что любое правильное решение оценивается в 7
баллов. Недопустимо снимать баллы за то, что решение слишком длинное,
или за то, что решение школьника отличается от приведенного в
методических разработках или от других решений, известных жюри.
В то же время любой сколь угодно длинный текст решения, не
содержащий полезных продвижений, должен быть оценен в 0 баллов.
1.3.4-277
Система оценивания результатов по физике
1.
Количество балов за каждую задачу теоретического тура лежит в
пределах от 0 до 10.
2.
Количество баллов за каждую задачу экспериментального тура может
лежать в пределах от 0 до 15 баллов в 10 и 11 классах.
3.
Если задача решена частично, то оценке подлежат этапы решения
задачи. Не рекомендуется вводить дробные баллы. В крайнем случае,
следует их округлять «в пользу ученика» до целых баллов.
4.
Не допускается снятие баллов за «плохой почерк» или за решение
задачи
способом,
не
совпадающим
со
способом,
предложенным
методической комиссией.
Примечание. Вообще не следует слишком догматично следовать авторской
системе оценивания (это лишь рекомендации!). Решения и подходы
школьников могут отличаться от авторских, быть не рациональными.
5.
Для облегчения решения задачи учащимися 9 – 11 классов и
унификации оценивания решенных задач, рекомендуется, если это
возможно, задавать в одной задаче несколько вопросов. В этом случае
оценка задачи получается суммированием баллов за ответы на каждый
вопрос, но не должна превышать указанную в п.п.1, 2.
Особое внимание надо обратить на применяемый математический
аппарат, используемый для задач, не имеющих альтернативных вариантов
решения. В первую очередь – понятия тригонометрии, квадратного корня,
(в 7-8 классах могут быть проблемы), стандартной формы записи числа (7
класс), экспонента и логарифм (9-10 класс), производная (10 класс),
интеграл (11 класс). В начале учебного года не все эти темы успевают
пройти.
Пример
соответствия
выставляемых
баллов
и
решения,
приведенного участником олимпиады.
1.3.4-278
Баллы
Правильность (ошибочность) решения
10
Полное верное решение
8-9
Верное решение. Имеются небольшие недочеты, в целом не
влияющие на решение.
Решение в целом верное, однако, содержит существенные
ошибки (не физические, а математические).
Найдено решение одного из двух возможных случаев.
6-7
5
Есть понимание физики явления, но не найдено одно из
необходимых для решения уравнений, в результате
полученная система уравнений не полна и невозможно найти
решение.
Есть отдельные уравнения, относящиеся к сути задачи при
отсутствии решения (или при ошибочном решении).
Решение неверное, или отсутствует.
3-4
1-2
0
Победители и призеры Олимпиады
Победители и призеры этапов Олимпиады определяются на
основании результатов участников соответствующих этапов Олимпиады.
Количество
соответствующему
победителей
каждого
общеобразовательному
этапа
Олимпиады
предмету,
по
по
которому
проводится олимпиада, не должно превышать 10 процентов от общего
числа
участников
соответствующего
этапа
Олимпиады
по
соответствующему общеобразовательному предмету. Общее количество
победителей
и
призеров
каждого
этапа
Олимпиады
по
общеобразовательному предмету, по которому проводится олимпиада, не
должно
превышать
соответствующего
35
процентов
этапа
от
Олимпиады
общего
по
числа
участников
соответствующему
общеобразовательному предмету.
Победители и призеры Олимпиады определяются по результатам
заключительного этапа Олимпиады. Победителями Олимпиады считаются
участники Олимпиады, награжденные дипломами 1 степени. Призерами
1.3.4-279
Олимпиады считаются участники Олимпиады, награжденные дипломами 2
и 3 степени.
Дипломы победителей и призеров Олимпиады подписываются
председателем оргкомитета Олимпиады.
Льготы победителям и призерам Олимпиады
При равенстве общего числа баллов преимущественным правом
поступления в Московский Государственный Технический Университет
Гражданской Авиации пользуются победители и призеры Олимпиады;
Победителям и призерам Олимпиады могут быть учтены результаты
Олимпиады
при
поступлении
в
Московский
Государственный
Технический Университет Гражданской Авиации по соответствующему
предмету только в текущем году набора.
Льготы победителям и призерам Олимпиады среди учащихся 10-х
классов предоставляются в виде возможности бесплатно пользоваться
библиотечными фондами, разрабатывать и испытывать новые дельталеты
в
Студенческом
конструкторском
бюро,
осваивать
компьютерные
программы в Центре новых информационных технологий, посещать
секции волейбола, баскетбола, футбола, настольного тенниса, легкой
атлетики, атлетической гимнастики и ритмической гимнастики, заниматься в
театральной и танцевальной (современный и народный танец) студии, студии
эстрадного и академического вокала, в вокально-инструментальных ансамблях
и коллективе КВН, студии художественного слова.
Разработка тестовых заданий и систем тестирования учащихся
при подготовке к олимпиадам
Одним из этапов подготовки к успешному написанию олимпиады
является решение тестовых заданий с несколькими вариантами ответов. К
тому же, такие задания могут служить для тестирования учащихся по
различным темам олимпиадных заданий. Отличным подспорьем для этого
1.3.4-280
является
сайт
http://intelmath.narod.ru/kangaroo-problems.html.
Задачи
собраны по темам. Сначала идут довольно легкие задачи, но постепенно их
сложность возрастает. Часть задач уже требует довольно глубоких и
основательных математических знаний.
Тема №1: комбинаторика, логика, взвешивания
Условия задач
Задача 1. Сколько существует наборов из двух или более
последовательных натуральных чисел, сумма которых равна 100?
А:1; Б:2; В:3; Г:4; Д:5.
Задача 2. Мы выписали все натуральные числа от однозначных до
семизначных, в записи которых используются только 0 и 1. Сколько
единиц мы записали?
А:128; Б:288; В:448; Г:512; Д:896.
Задача 3. Чему равняется
А:
; Б:
; В:
; Г:
?
; Д: ;
.
Задача 4. В каждом из пяти стаканов кофе, какао или молоко. Общий
объём кофе вдвое больше объёма какао. Известно, что ни в каких трёх
стаканах нет одинакового напитка. В каком стакане какао?
Задача 5. Детская игрушка подвешена к потолку и находится в
равновесии. Одинаковые фигурки весят одинаково. Шарик весит 30
граммов. Сколько весит кубик, отмеченный знаком вопроса?
1.3.4-281
А:10г; Б:20г; В:30г; Г:40г; Д:50г.
Решения
Задача 1. Сначала заметим, что суммой двух последовательных
чисел число 100 не получить, т.к. из двух последовательных, одно будет
чётным, а другой нечётным, и их сумма обязательно будет нечётным
числом. Если же сложить 3 последовательных натуральных числа:
n+n+1+n+2, то их сумма 3n+3 будет обязательно делиться на 3,
следовательно, и здесь мы число 100 не получим. Для четырёх слагаемых
окажется, что их сумма n+n+1+n+2+n+3=4n+6 даёт остаток 2 при делении
на 4, а 100 делится на 4 нацело, итак, и здесь неудача.
Рассмотрим в общем случае сумму k последовательных натуральных
чисел.
n+n+1+n+2+...+n+k-1=kn+k(k-1)/2
Отсюда видно, что при нечётных k сумма будет делиться на k, а при
чётных k сумма k последовательных натуральных чисел будет давать
остаток k/2 при делении на k, и, следовательно, будет делиться на k/2.
Выясним теперь, какое наибольшее количество последовательных
слагаемых может образовать сумму в 100. Поскольку наименьшая по
величине сумма из 14-ти слагаемых 1+2+...+14=14*15/2=105>100, то
достаточно перебрать все k, не большие 13-ти. Из нечётных подходит
только 5, т.к. на 7, 9, 11 или 13 число 100 не делится. Из чётных подходит
только 8 (100=12*8+4).
Можно привести и соответствующие разбиения:
1.3.4-282
100=18+19+20+21+22
100=9+10+11+12+13+14+15+16
Итак, получили 2 способа.
Ответ. Б:2 способа.
Кстати, интересно найти трёхзначное число, которое можно
представить в виде суммы нескольких последовательных натуральных
чисел наибольшим количеством способов.
Задача 2. Хмм… интересно. Цифры 0 и 1, а также варианты ответа
А:128 и Г:512 наталкивают на мысль об использовании двоичной системы
счисления. И вправду, задачу можно переформулировать так: сколько
единиц используется в двоичной записи всех чисел от 1 (даже от 0, на
ответ это не повлияет) до 127? (от 0000000 до 1111111?)
Но ведь из этих 128-ми чисел в каждом из семи двоичных разрядов
ровно у половины стоит единица, у другой половины – 0. Имеем 7*64=448
единиц во всех числах.
Ответ. В: 448.
Задача 3. Первый вариант ответа
заставляет
вспомнить о правильно-неправильных сокращениях и выносах из корня,
когда выполняя неверные с математической точки зрения действия мы
получаем верный результат. Однако вряд ли здесь это. Вариант
больше похож на правду, учитывая, что для близких к
данным в условии числам 300, 400 и 500 равенство выполняется
(увеличенный в 100 раз египетский треугольник).
Стоп! А если мы египетский треугольник увеличим не в 100, а в 101
раз, что получим? Как раз эту тройку: 303, 404, 505. Значит, окончательно,
ответ В:
1.3.4-283
Задача 4. Вопрос звучит «В каком стакане какао?», значит, стакан с
какао один. Тогда в двух из остальных четырёх стаканов кофе, и в двух –
молоко.
В первом стакане какао быть не может, т.к. его объём максимальный
и 2 других стакана не смогут занимать вдвое больший объём. А второй
стакан (750г) подходит, тогда кофе будет в первом и третьем стаканах
(950+550). Поскольку тест предполагает однозначный ответ, на этом
можно и остановиться, сэкономив драгоценное время на решение других
задач. Нам же с вами можно спокойно посидеть и убедиться, что
действительно ни для какого из оставшихся стаканов нельзя найти двух
других таких, чтобы они занимали вдвое больший объём.
Ответ. Б.
Задача 5. Обратим внимание, что в формулировке задачи не
говорится
стандартная
фраза
«весом
самой
конструкции
можно
пренебречь». А ведь она и действительно не нужна – все перекладины
уравновешивают друг друга и на решение не влияют.
Из правой части заметим, что трапеция равна по весу двум шарикам.
Из левой части одно сердечко равно по весу двум кубикам.
Значит, 6 кубиков равны четырём шарикам. 6 кубиков весят 120г, значит 1
кубик весит 20г.
Ответ. Б.
Тема №2: комбинаторика, логика, выражения
Условия задач
Задача 6. На гранях куба написаны некоторые натуральные числа, и
у каждой вершины написано число, равное произведению чисел на гранях,
прилежащих к этой вершине. Сумма чисел на вершинах равна 100. Тогда
наибольшая возможная сумма чисел на гранях равна:
1.3.4-284
А:14; Б:17; В:25; Г:29; Д:100.
Задача 7. Рассмотрим множество всех чисел, которые состоят из
цифр 1, 2, 3, 4 без повторов. Чему равна сумма всех этих чисел?
А:5550; Б:99990; В:66660; Г:100000; Д:98760.
Задача 8. Чему равняется первая цифра наименьшего натурального
числа, сумма цифр которого равна 2001?
А:1; Б: 2; В: 3; Г: 4; Д: 5.
Задача 9. M, D, S, E, K сидят на скамейке в парке. М не сидит справа
на краю, а D не сидит слева на краю. S не сидит на краю. K не сидит рядом
с S, а S не сидит рядом с D. E сидит справа от D, но не обязательно рядом.
Кто сидит крайним справа?
А:Невозможно определить; Б: D; В: S; Г: E; Д: K.
Задача 10. Ира, Аня, Катя, Оля и Эля живут в одном доме: две
девочки на первом этаже и три на втором.. Оля живёт не на том этаже, где
Катя и Эля. Аня - не на том этаже, где Ира и Катя. Кто живёт на первом
этаже?
А:Катя и Эля; Б:Ира и Эля; В:Ира и Оля; Г:Ира и Катя; Д:Аня и Оля.
Решения
Задача 6. Пусть числа на верхней и нижней гранях куба равны a и b,
а на его боковых гранях: c, d, e и f. Тогда сумма чисел у вершин куба будет
равна:
acd+ade+aef+acf+bcd+bde+bef+bcf=
=a(cd+de+ef+cf)+b(cd+de+ef+cf)=
=(a+b)(cd+de+ef+cf)=(a+b)(c(d+f)+e(d+f))=(a+b)(c+e)(d+f)=100.
Получаем произведение трёх натуральных чисел, каждое из которых
не менее двух, равно 100. Число 100 в виде такого произведения можно
представить тремя способами. 100=2*2*25=2*5*10=4*5*5. Из них сумма
1.3.4-285
множителей, а, соответственно, и сумма a+b+с+d+e+f наибольшее
значение принимает в первом случае.
Ответ. Г: 29.
Задача 7. Всего таких чисел будет 4!=24. При этом в любом из
четырёх разрядов ровно у шести из них будет стоять 1, у шести - 2, у шести
- 3 и у шести чисел - 4. Искомую сумму можно представить как:
6(1000*(1+2+3+4)+100*(1+2+3+4)+10*(1+2+3+4)+(1+2+3+4))=66660.
Ответ. В: 66660.
Задача 8. Из всех числе с такой суммой цифр наименьшим будет вопервых число, состоящее из наименьшего количества цифр, а во-вторых, с
наименьшей возможной первой цифрой. Сколько знаков должно быть у
числа, чтобы сумма его цифр могла составлять 2001? 2001/9=222 (остаток
3). Значит наименьшим числом с суммой цифр 2001 будет число 399...9,
где за тройкой идёт 222 девятки.
Ответ. В: 3.
Задача 9. Для удобства занумеруем места слева направо: 1, 2, 3, 4, 5
и занумеруем утверждения задачи: М не сидит справа на краю(I), а D не
сидит слева на краю(II). S не сидит на краю(III). K не сидит рядом с S(IV),
а S не сидит рядом с D(V). E сидит справа от D, но не обязательно
рядом(VI). Теперь из I, III VI определяем, что на 5 месте не могут сидеть
M, S или D. Из II, III, VI определяем, что на 1м месте не могут сидеть D, S
или E. Таким образом, из V, для расположения S и D есть только два
варианта: S2, D4 или D2, S4. Но в первом случае 5е место должно быть
занято Е, и тогда К не удаётся усадить не рядом с S, согласно IV. Для
второго варианта однозначно K1, M3, E5. Итак, справа сидит Е.
Ответ. Г: Е.
Задача 10. Получается, что Ира и Катя, а также Катя и Эля живут на
одном этаже. Значит, они живут на втором. На первом этаже тогда живут
Аня и Оля.
1.3.4-286
Ответ. Д: Аня и Оля.
Тема №3: взвешивания, логика, делимость
Условия задач
Задача 11. Какое наименьшее количество гирь необходимо, чтобы
иметь возможность взвесить на чашечных весах любой груз массой от 1 до
10г? (Масса выражается только целым числом граммов и гири можно
класть на обе чаши весов).
А:2; Б:3; В:4; Г:5; Д:10.
Задача 12. Сумма цифр натурального числа m равна 30. Чему не
может равняться сумма цифр числа (m+3)?
А:6; Б:15; В:21; Г:24; Д:33.
Задача 13. В сумке более одного кенгуру. Первый кенгуру сказал
"Нас здесь шестеро", – и выпрыгнул из сумки. Затем через каждую минуту
один из оставшихся кенгуру говорил "Все, кто выпрыгнул передо мной,
говорили неправду",- и также выпрыгивал. Сколько кенгуру сказали
правду?
А:0; Б: 1; В: 2; Г: 6; Д: все.
Задача 14. Если кенгуру при прыжке оттолкнётся левой ногой, то
прыгнет на 2 метра. Если оттолкнётся правой ногой, то длина прыжка
составит 4м. Если же обеими ногами, то прыгнет на 7 метром. Какое
наименьшее количество прыжков должен сделать кенгуру, чтобы
проскакать ровно 1000м?
А:142; Б: 144; В: 250; Г: 500; Д: другой ответ.
Задача 15. В компании из пяти человек есть вруны, которые всегда
говорят неправду, и честные, которые всегда говорят правду. Каждого из
них спросили: "Сколько врунов в вашей компании?", на что были
1.3.4-287
получены ответы: "один", "два", "три", "четыре" и "пять". Сколько на
самом деле врунов в этой компании?
А:1; Б:2; В:3; Г:4; Д:5.
Решения
Задача 11. Запишем числа от 1 до 10 в двоичной системе: 0001, 0010,
0011, 0100, 0101, 0110, 0111, 1001, 1010. Поскольку в них не более четырёх
двоичных разрядов, для взвешивания этих грузов можно обойтись
четырьмя гирьками, массой 1г, 2г, 4г и 8 г. Но здесь мы не используем
возможность класть гирьки на обе чаши весов. А в таком случае более
экономной будет троичная форма записи, но с цифрами не 0, 1, 2, а с (-1),
0, 1:
1=1=001
2=3-1=01(-1)
3=3=010
4=3+1=011
5=9-3-1=1(-1)(-1)
6=9-3=1(-1)0
7=9-3+1=1(-1)1
8=9-1=10(-1)
9=9=100
10=9+1=101
Всего же с помощью гирек массой 1г, 3г и 9г можно взвесить грузы до
9+3+1=13г.
Ответ. Б: 3.
Задача 12. Первая идея: поскольку сумма цифр числа m равна 30,
оно делится на 3, значит и (m+3) делится на 3, делиться должна и сумма
его цифр. Однако этим свойством обладают все предложенные варианты.
Копнём глубже: т.к. сумма цифр числа m равна 30, его остаток от деления
1.3.4-288
на 9 равен остатку от деления на 9 числа 30, т.е. трём. Значит число (m+3),
как и его сумма цифр, должно давать остаток 6 при делении на 9. Такой
остаток дают все предложенные варианты, кроме В:21. Он-то и будет
невозможным.
Ответ. В: 21.
Задача 13. Если в сумке шестеро кенгуру, то первый сказал правду, а
все остальные своим утверждением: "Все, кто выпрыгнул передо мной,
говорили неправду",– соврали. Если же в сумке сидело не 6 кенгуру, то
соврал первый, второй сказал правду, а остальные (если они были), снова
соврали. В любом случае, правду сказал только один.
Ответ. Б: 1.
Задача 14. Определим, за сколько прыжков можно максимально
приблизиться к отметке 1000. За 142 прыжка можно продвинуться на 994
метра.
Оставшиеся
6
метров
можно
преодолеть
за
2
прыжка:
оттолкнувшись последовательно левой и правой ногой. Кстати, если бы
кенгуру умел прыгать и на 1 метр, то в 144 прыжка можно было уложиться
и другим способом: 143 прыжка по 7 метром и затем на 1 метр назад.
Ответ. Б: 144.
Задача 15. Если бы честных в компании было несколько, то мы бы
получили как минимум 2 одинаковых ответа (однако наличие одинаковых
ответов не гарантирует того, что они принадлежат честным, ведь и вруны
могли одинаково соврать). В нашем случае 5 врунов быть не может,
поскольку тот, кто сказал “пять”, сказал бы правду. А вот 4 вруна – вполне
возможно.
Ответ. Г: 4.
1.3.4-289
Тема №4: делимость, последовательности, рассуждения
Условия задач
Задача 16. В озере плавает яблоко: 2/3 его под водой и 1/3 – над
водой. К нему подплывает рыба и подлетает птица, и одновременно
начинают его есть. Птица есть вдвое быстрее, чем рыба. Какую часть
яблока съест птица?
А:1/3; Б:1/2; В:2/3; Г:3/5; Д:4/5.
Задача 17. Если зачеркнуть последнюю цифру натурального числа,
оно уменьшится в 14 раз. Сколько существует натуральных чисел с таким
свойством?
А:0; Б:1; В:2; Г:3; Д:4.
Задача 18. Первый элемент последовательности равен 2, второй
равен 3. каждый элемент, начиная со второго, на 1 меньше произведения
предыдущего и следующего элементов. Чему равна сумма первых 2003
элементов этой последовательности?
А:2358; Б:2989; В:3241; Г:3607; Д:3745.
Задача 19. Известно, что “микс” 36 равен 18, “микс” 325 – 30, “микс”
45 – 20, “микс” 30 равен 0. Найдите “микс” 531.
А:10; Б:15; В:16; Г:21; Д:22.
Задача 20. Красная Шапочка несла бабушке 14 пирожков: с мясом,
грибами и капустой. Пирожков с капустой было больше всего, их было
вдвое больше, чем пирожков с мясом, а пирожков с мясом было больше,
чем пирожков с грибами. Сколько пирожков с грибами несла Красная
Шапочка?
А:2; Б:4; В:5; Г:1; Д:3.
1.3.4-290
Решения
Задача 16. Вспоминается физический опыт “водяной подсвечник”.
Если в нижнюю часть парафиновой свечи воткнуть гвоздь так, чтобы та
плавала в стакане вертикально, то по мере сгорания свеча будет
подниматься, но вес воды, вытесненной погружённой частью всегда будет
равен общему весу свечи. Здесь так же: по мере исчезновения в желудках
существ, всё равно 1/3 яблока будет над водой, а 2/3 – под водой. И,
поскольку птица есть вдвое быстрее, она в конечном итоге съест 2/3
яблока, а рыба – 1/3.
Ответ. В: 2/3.
Задача 17. Пусть последняя цифра числа равна х, а число с
зачёркнутой последней цифрой равно у. Тогда 14у=10у+х, откуда х=4у.
Поскольку х – цифра, то может быть 3 варианта:
х=0, у=0, но если в числе 0 зачеркнуть 0 не останется вообще ничего.
х=4, у=1. Получаем 14=1*14.
х=8, у=2. Получаем 28=2*14.
Ответ. В: 2.
Задача
18.
Построим
несколько
элементов
данной
последовательности в соответствии с правилом. 2, 3, 2, 1, 1, 2, 3, 2, 1, 1 ...
Можно заметить, что в последовательности повторяется группа из пяти
элементов: 2, 3, 2, 1, 1, их сумма равна 9. А среди 2003 последовательных
элементов такая группа встретится 400 раз, затем пойдёт числа 2, 3, 2.
Искомая сумма будет равна 400*9+2+3+2=3607.
Ответ. Г: 3607.
Задача 19. На основе данных примеров можно догадаться, что
“микс” числа – это произведение всех его цифр. Поэтому “микс”
531=5*3*1=15.
Ответ. Б: 15.
1.3.4-291
Задача 20. Так как пирожков с капустой вдвое больше, чем с мясом,
то их общее количество должно делиться на 3. Такое будет возможно
только когда пирожков с грибами 2 или 5. Но если тех будет 5, то
пирожков с мясом должно быть 4, что не соответствует второму условию.
Значит. С грибами было всего 2 пирожка.
Ответ. А: 2.
Тема №5: вероятность, геометрия, логика
Условия задач
Задача 21. Случайным образом в решётке 3х4 точек выбрали три из
них.
*
*
*
*
*
*
*
*
*
*
*
*
Какова вероятность того, что эти точки будут лежать на одной
прямой?
А:1/12; Б:1/11; В:1/16; Г:1/8; Д:1/4.
Задача 22. Квадрат 4х4 разделили на 16 единичных квадратов. Найти
максимально возможное количество диагоналей, которые можно провести
в этих единичных квадратах так, чтобы они не имели общих точек
(включая концы).
А:8; Б:9; В:10; Г:11; Д:12.
Задача 23. Пусть m – произведение периметра треугольника на
сумму трёх высот этого треугольника. Какое из высказываний ложно, если
площадь этого треугольника равна 1?
А:m может быть больше 1000; Б:всегда m>6; В:m может равняться 18;
Г:если треугольник правильный, то m>16; Д:m может быть меньше 12.
1.3.4-292
Задача 24. Какое наибольшее количество цифр можно стереть в
1000-значном числе 20082008…2008, так, чтобы сумма оставшихся цифр
равнялась 2008?
А:260; Б:510; В:746; Г:254; Д:130.
Задача 25. В коробке лежат 7 карточек с написанными на них
числами от 1 до 7 (по одному числу на карточке). Первый мудрец наугад
берёт три карточки из коробки, а второй – две (ещё две карточки остаются
в коробке). Первый мудрец, глядя на свои карточки, говорит второму: «Я
точно знаю, что сумма чисел на твоих карточках чётная». Сумма чисел,
записанных на карточках первого мудреца, равняется:
А:6; Б:9; В:10; Г:12; Д:15.
Задача 26. Котик-Муркотик и Лисичка-Сестричка ловили рыбу. К
ним подбежал голодный Волчик-Братик и спросил, много ли рыбы они
поймали? Лисичка хитро ответила: у нас двоих рыб на 7 больше, чем у
меня одной, а у одного из нас на 17 рыб меньше, чем у другого. Сколько
рыбы словили вместе Котик-Муркотик и Лисичка-Сестричка?
А:10; Б:17; В:22; Г:27; Д:31.
Решения
Задача 21. Подсчитаем, сколько троек точек могут образовать
прямую линию. В каждом из трёх горизонтальных рядов таких троек будет
по 4, также такими тройками будут 4 вертикальных и 4 диагональных ряда
точек. Таким образом, всего троек точек, удовлетворяющих условию будет
20. Всего способов выбрать 3 точки из 12 будет 220. Таким образом,
искомая вероятность равна 20/220=1/11.
Ответ. Б: 1/11.
Задача 22. Раскрасим столбцы вершин единичных квадратов чёрным
и белым цветами. Каждая диагональ единичного квадрата будет иметь
1.3.4-293
концами точки разного цвета. Поэтому непересекающихся диагоналей
может быть не более десяти. Один из вариантов представлен ниже.
Ответ. В: 10.
Задача 23. Поскольку
и
, то
. Тогда
,
. Отсюда сразу видно,
что если взять треугольник с площадью 1 и достаточно большим
периметром, величина m может превысить 1000. Если раскрыть скобки, то
получиться
большие
выражение
шести.
Для
вида
m=6+f(a,b,c),
правильного
принимающее
треугольника
значения,
m=3*2*3=18>16.
Следовательно, методом исключения, неверно пятое утверждение.
Ответ: Д:m может быть меньше 12.
Задача 24. В данном числе последовательность 2008 повторяется
1000/4=250 раз. Сумма цифр этого числа равна 2500. Если вытереть 500
нулей, она не изменится. Таким образом, варианты А, Г и Д отметаем
сразу, а вариант Б отметается после следующего размышления: даже если
дополнительно вычеркнуть 10 восьмёрок, сумма оставшихся цифр
уменьшится лишь на 80. Поэтому остаётся только вариант В, в целях
экономии времени на олимпиаде его можно не проверять. А вообще,
действительно: зачёркиваем 500 нулей и 246 двоек и сумма оставшихся
цифр будет равна 2008.
Ответ. В: 746.
1.3.4-294
Задача 25. Первый мудрец будет точно знать, что сумма цифр на
карточках старого мудреца чётна лишь в том случае, если ему будет
известно, что все оставшиеся числа – одной чётности. Следовательно, ему
могли выпасть только карточки с числами 2, 4 и 6, их сумма равна 12.
Данная задача проходила в 2008 году сквозь все классы и была
использована
для
мониторинга
умения
совершать
логические
умозаключения. Наряду со стабильным ростом из класса в класс, тем не
менее был отмечен общий низкий уровень данного умения.
Ответ. Г: 12.
Задача 26. Если у них двоих рыб на 7 больше, чем у Лисички, то эти
7 рыб принадлежат Коту. И выходит, что у Лисички рыб на 17 больше, т.е.
24. Итого 31 рыба.
Ответ. Д: 31.
Тема №6: целая и дробная часть, делимость, цифры
Условия задач
Задача
27.
При
каком
выполняется неравенство
наибольшем
значении
параметра
a
?
А:1; Б:2; В:3; Г:4; Д:5.
Задача 28. Из какого набора цифр, приведённых далее, состоит
семизначное число, если оно делится на каждую из своих цифр и все его
цифры различны?
А:0123467; Б:1246789; В:1235679; Г:2356789; Д:1236789.
Задача 29. Сколько номеров лет ХХ века можно представить в виде
разности двух степеней двойки?
А:0; Б:1; В:2; Г:3; Д:4.
Задача 30. У Тани в коробке 9 карандашей. Как минимум один из
них синего цвета. Среди каждых 4 карандашей как минимум два –
1.3.4-295
одинакового цвета, а среди каждых пяти не более трёх одинакового цвета.
Сколько синих карандашей у Тани в коробке?
А:1; Б:2; В:3; Г:4; Д: Невозможно определить.
Задача 31. На счётчике пробега моей машины сейчас показано число
187369 (км). В этом числе все цифры различны. Какое наименьшее
количество километров нужно проехать, чтобы на счётчике опять
появилось число, у которого все цифры различны?
А:1; Б:21; В:431; Г:12431; Д:13776.
Решения
Задача 27. Понятно, что при x<0 неравенство выполняется при
любых значениях параметра a. Для нуля равенство также будет иметь
место всегда. А для положительных значений x используем тот факт, что
. Тогда неравенство можно представить в виде
откуда
,
. А т.к. сумма квадратов двух чисел не менее
удвоенного их произведения, то a-2 не превосходит двух, отсюда
наибольшее значение параметра a – 4.
Ответ. Г: 4.
Задача 28. Первым делом отпадает вариант А, как содержащий
цифру 0. Затем отбрасываем варианты В (1235679) и Г (2356789),
содержащие одновременно двойку и пятёрку. Основание следующее:
число должно одновременно делиться на 2 и на 5, следовательно, на 10, и
заканчиваться нулём. А нуля среди этих наборов цифр нет, да и быть не
может в наборе, удовлетворяющем условиям задачи.
Из оставшихся двух наборов сделать выбор поможет признак
делимости на 9, который инвариантен расположению цифр. Сумма цифр в
наборе Б на 9 не делится, хотя девятка в нём присутствует. Следовательно,
искомый набор – Е (1236789). Поскольку задачи найти конкретно число,
1.3.4-296
обладающее данным свойством, не ставилось, на этом можно было и
успокоиться. Однако мне стало интересно и с помощью переборной
программы я обнаружил, что таких чисел целых 105, начиная с 1289736 и
заканчивая 9867312. А несколько подумав, можно и без перебора
убедиться, что для восьмизначных чисел не существует аналогичных
наборов цифр. Вообще, интересно было бы рассмотреть такую задачу в
других системах счисления, вполне возможно, что это будет темой одной
из статей.
Ответ. Е: 1236789.
Задача 29. Пусть для некоторого числа x выполняется равенство
, тогда
. Следовательно,
и может
принимать лишь k различных значений. Поскольку промежуток [1901;
2000] принадлежит промежутку [2048; 1024], то значение k=11. А
промежутку [1901; 2000] будут принадлежать только две из одиннадцати
разностей: 1984=2048-64 и 1920=2048-128.
Ответ. В: 2.
Задача 30. Поскольку среди каждых пяти карандашей не более трёх
одноцветных, то и во всём наборе количество карандашей одинакового
цвета не превышает трёх. Следовательно, т.к. карандашей 9, то и
количество цветов не менее трёх. Однако, если бы карандаши у Тани были,
к примеру, четырёх цветов, то взяв по одному карандашу каждого цвета
мы получили бы противоречие с первым условием. Следовательно, у Тани
карандаши ровно по 3 карандаша трёх различных цветов, в том числе и
синих.
Ответ. В: 3.
Задача 31. Эта задача – хороший пример приёма перебора в тесте.
Вместо того, чтобы непосредственно вычислять, какое расстояние нужно
проехать, прежде чем вновь появится число из разных цифр, будет
1.3.4-297
последовательно пытаться прибавлять к 187369 числа из вариантов ответа
и смотреть, что получится.
187369+1=187370 – не подходит
187369+21=187390 – подошло!
Ответ. Б: 21.
Тема №7: комбинаторика, среднее арифметическое, дроби
Условия задач
Задача
32.
Сколькими
способами
можно
прочесть
слово
KANGAROO, двигаясь только вниз и вправо?
KANGAROO
ANGAROO
NGAROO
GAROO
AROO
ROO
OO
O
А:168; Б:224; В:128; Г:256; Д:328.
Задача 33. Сколько существует таких четырёхзначных чисел, у
которых сумма двух последних цифр и числа, образованного двумя
первыми цифрами, равняется числу, образованному двумя последними
цифрами? (Пример числа, удовлетворяющего данному условию: 6370, т.к.
7+0+63=70).
А:10; Б:45; В:50; Г:80; Д:90.
Задача
34.
Среднее
арифметическое
десяти
различных
положительных целых чисел равняется 10. Чему может равняться
наибольшее среди этих чисел?
1.3.4-298
А:10; Б:45; В:50; Г:55; Д:91.
Задача 35. В селе Кенгуровка есть две улицы: Яблочная и Грушёвая.
Половина всех домов села расположены на Яблочной улице, а четверть –
на Грушёвой. У каждого дома четыре окна: да белых, синее и красное.
Каких окон больше: красных на Яблочной или белых на Грушёвой?
А: одинаково; Б: красных вдвое больше, чем белых; В: белых вдвое
больше, чем красных; Г: невозможно определить; Д: ответ зависит от
количества синих окон.
Задача 36. Если мы умножим число 12345679 на 9, то получим число
111111111. Если мы умножим его на 18, то получим результат, который
содержит только цифры 2. Если мы умножим это число на 27, то получим
число, которое записывается только при помощи цифры 3. На какое число
нужно умножить число 12345679, чтобы получить число из одних
семёрок?
А:43; Б:53; В:63; Г:73; Д:83.
Решения
Задача 32. Прочитав очередную букву, имеем 2 варианта: повернуть
или вправо, или вниз. Т.к. всего возможных повторов будет 7, то слово
KANGAROO можно прочесть 128-ю способами. Параллельно можно
заметить, что если подписать на каждой букве количество способов,
которыми до неё можно дойти, то получим повёрнутый на бок треугольник
Паскаля.
Ответ. В: 128.
Задача
33.
Пусть
четырёхзначное
число
имеет
вид
1000a+100b+10c+d, где a, b, c, d – натуральные числа и a>0. Тогда условие,
выполнение которого требуется, можно записать как: c+d+10a+b=10c+d.
Отсюда получаем: 10a+b=9c. Следовательно, последняя цифра может быть
любой, а число, образованное первыми двумя цифрами, должно быть в 9
1.3.4-299
раз больше третьей цифры. Значит третья цифра может быть равной 2, 3,
… 9 (т.к. a>0), а четвёртая – 0, 1, … 9. Всего 8*10=80 вариантов.
Ответ. Г: 80.
Задача 34. Если среднее арифметическое десяти чисел равно 10, то
их сумма равна 100. Десятое число будет наибольшим из возможных,
когда остальные девять чисел будут наименьшими из возможных.
Поскольку они все различны, то это должны быть 1, 2, … 9, с суммой
равной 45. Следовательно максимальное значение десятого числа равно 55.
Ответ. Г: 55.
Задача 35. На Яблочной улице домов вдвое больше, чем на
Грушёвой. А в каждом доме белых окно вдвое больше, чем красных.
Значит красных окон на Яблочной улице ровно столько же, сколько и
белых на Грушёвой.
Ответ. А: одинаково.
Задача 36. Чтобы получить число из одних семёрок, нужно
умножить 111111111 на 7, а это то же самое, если умножить 12345679 на
9*7=63. Кстати, на свойствах 12345679, “числа без восьмёрки”, основано
ещё несколько интересных математических трюков и фокусов. А чтобы
понять, что в нём такого особенного, разделите 1 на 81.
Ответ. В: 63.
Тема №8: числовые процессы, выбор, множества
Условия задач
Задача 37. Сколько существует непустых подмножеств множества
{1, 2, 3, ..., 12}, в которых сумма наибольшего и наименьшего элементов
равна 13?
А: 1024; Б: 1175; В: 1365; Г: 1785; Д:4095.
1.3.4-300
Задача 38. Числа 1, 2, 3 записаны по кругу. Затем между ними
поместили суммы пар соседних чисел. Получили 6 чисел: 1, 3, 2, 5, 3, 4.
Эту операцию повторили ещё 4 раза и в результате получили 96 чисел.
Чему равна сумма этих 96-ти чисел?
А: 486;Б:998; В: 1458; Г: 2187; Д: 4374.
Задача 39. Мама попросила маленького Ваню рассортировать
парами его носки после стирки. Но он бросил носки в комод, не сортируя.
Там было 5 пар чёрных, 10 пар коричневых и 15 пар серых носков. Ваня
собирается пойти в поход на 7 дней. Какое минимальное количество носок
ему нужно вытащить из комода, чтобы среди них наверняка оказались 7
пар одного цвета?
А: 21;Б:31; В: 37; Г: 40; Д: 41.
Задача 40. С полудня до полуночи Ученый Кот спит под дубом, а с
полуночи до полудня он рассказывает сказки. Табличка на дубе говорит:
«Два часа назад Учёный Кот делал то же, что он будет делать через час».
Сколько часов в сутки табличка говорит правду?
А: 3;Б:6; В: 12; Г: 18; Д: 21.
Задача 41. Марк загадал трёхзначное и двузначное числа, разность
которых равна 989. Тогда сумма этих чисел равна:
А: 1000;Б: 1001; В: 1009; Г: 1010; Д: 2005.
Решения
Задача
подмножество
37.
Из
множества
в
n
элементов
можно
выбрать
способами. Таким образом, множеств с наибольшим
элементом 12 и наименьшим элементом 1 будет столько, сколько
подмножеств множества {2, 3, ..., 10, 11}, а именно 1024. Далее, множеств
с наибольшим элементом 11 и наименьшим элементом 2 будет столько,
сколько подмножеств множества {3, 4, ..., 9, 10}, а именно 256. Далее
будет ещё 64 множества с наибольшим элементом 10 и наименьшим
1.3.4-301
элементом 3, 16 – с наибольшим 9 и наименьшим 4, 4 с наибольшим 8 и
наименьшим 5 и одно с наибольшим элементом 7 и наименьшим 6. Всего
1024+256+64+16+4+1=1365.
Ответ. В: 1365.
Задача 38. Поскольку каждое из дописываемых чисел равно сумме
двух соседних чисел из предыдущей итерации, то сумма всех новых чисел
будет равна удвоенной сумме всех чисел, бывших на окружности до
дописывания. Таким образом, после каждой операции дописывания сумма
всех чисел на окружности утраивается, а их количество удваивается.
Значит, после 5-ти дописываний изначальная сумма чисел, равная 6,
увеличится в 243 раза и станет равной 1458.
Ответ. В: 1458.
Задача 39. При решении таких задач нужно рассматривать самый
неблагоприятный вариант. А именно: какое максимальное количество
носок можно вытащить так, чтобы среди них не оказалось 7 пар одного
цвета? Ясно, что это будет возможным, если мы вытащим 10 чёрных, 13
коричневых и 13 серых носок: всего 36. Значит, если вытащить 37 носок,
то среди них гарантированно будет 7 пар одного цвета.
Ответ. В: 37.
Задача 40. Поскольку табличка охватывает временной диапазон в 3
часа, то надпись на ней станет правдивой в 2 часа дня и перестанет – в 11
часов вечера. Затем снова станет правдивой в 2 часа ночи и перестанет – в
11 утра. Всего 18 часов.
Ответ. Г: 18.
Задача 41. Число 989 может получиться только если от наибольшего
трёхзначного отнять наименьшее двузначное число: 989=999-10. Значит,
сумма этих чисел равна 999+10=1009.
Ответ В:1009.
1.3.4-302
Тема №9: комбинаторика, проценты, ребус
Условия задач
Задача 42. Сколько существует 10-значных чисел, состоящих только
из цифр 1, 2 и 3 таких, в которых соседние цифры отличаются на 1?
А: 16; Б: 32; В: 64; Г: 80; Д:100.
Задача
43.
На
выборах
мера
города
Кенгуруполя
было
зарегистрировано 2 кандидата. После обработки n% бюллетеней для
голосования избирательная комиссия сообщила жителям, что кандидат А
набрал 62% голосов, а кандидат В – 38% голосов. При каком минимальном
целом n эти предварительные результаты выборов гарантируют победу
кандидату А, если недействительных бюллетеней не будет? Мер
избирается простым большинством.
А: 55;Б:62; В: 81; Г: 87; Д: 93.
Задача 44. В уравнении K+A+N+G+A+R+O+O=56 разные буквы
обозначают разные цифры, а одинаковые буквы – одинаковые цифры.
Тогда значение суммы A+O равняется:
А: 18;Б:17; В: 16; Г: 15; Д: однозначно определить невозможно.
Задача 45. Дано 4 утверждения о натуральном числе А:
А делится на 5, А делится на 11, А делится на 55, А меньше 10. Известно,
что два из них правильные, а другие два – неправильные. Тогда А
равняется:
А: 0;Б:5; В: 10; Г: 11; Д: 55.
Задача
46.
Маша
коллекционирует
фотографии
известных
спортсменов. Количество фотографий, которые она собирает за каждый
год равно количеству фото, собранных за два предыдущих года. В 2008
году она собрала 60 фотографий, а в этом – 69. Сколько фотографий
собрала Маша в 2006 году?
А: 20;Б: 24; В: 36; Г: 40; Д: 48.
1.3.4-303
Задача 47. Серёжа подбрасывал игральный кубик четыре раза и
каждый раз записывал полученное число очков. Сложив эти числа, он
получил 21 очко. Какое наибольшее количество раз могла выпадать
тройка?
А: 0;Б: 1; В: 2; Г: 3; Д: 4.
Решения
Задача 42. В записи таких чисел после единиц и троек обязательно
должна идти двойка. После двоек может идти как единица, так и тройка.
Таким образом, если первая цифра числа - 1, то вторая определяется
однозначно, третья может быть одной из двух, четвертая – вновь
однозначно, пятая – одна из двух вариантов и т.д. Получается
чисел,
начинающихся на единицу. Столько же чисел будет начинаться на 3. Если
же первой цифрой будет двойка, то вторая цифра – одна из двух, третья –
однозначно и т.д., всего
вариантов. Итого 16+16+32=64 числа.
Ответ. В: 64.
Задача 43. Следует рассмотреть самый неблагоприятный для А
случай – все оставшиеся неподсчитанными бюллетени были поданы за
кандидата В. Тогда за А подано 0,62n% голосов, а за В – 0,38n+100-n =
100-0,62n процентов. Решая неравенство 0,62n>50 получаем, n>80,65.
Минимальное целое n, удовлетворяющее этому неравенству равно 81.
Ответ. В: 81.
Задача 44. Сразу видно, что ответ 18 в этом числовом ребусе
невозможен, т.к. цифры А и О различны. Проверим 17=9+8. Запишем
уравнение ребуса так:
2(A+O)=56-(K+N+G+R)
Наименьшее значение правой части будет, если за согласными
буквами будут спрятаны числа 7, 6, 5, 4. Тогда 2(8+9)=34=56-(7+6+5+4).
Сходится. Однозначный ли это ответ? Если А+О=16=9+7, то значение 561.3.4-304
(K+N+G+R) не может быть меньше, чем 56-(8+6+5+4)=33. А нужно
получить 32. И далее, снижая на 1 сумму А+О, значение выражения
2(А+О) снижается на 2, а правая часть уравнения нельзя будет уменьшить
менее, чем на 1, и она будет отставать. Значит А+О=17 – однозначное
решение данного числового ребуса.
Ответ. Б: 17.
Задача 45. Если правильны первые 2 утверждения, автоматически
становится верным и третье. И обратно: если верно третье (про делимость
на 55), то отсюда следует, что А делится и на 5, и на 11. Значит из первых
трёх утверждений верно лишь одно. Значит, верно и четвёртое
утверждение: A<10. А т.к. нет натуральных чисел, меньших 10 и
делящихся на 11, то верным будет первое: А делится на 5. Следовательно,
А=5.
Ответ. Б: 5.
Задача 46. В 2007 Маша должна была собрать 36 фотографий, чтобы
в сумме с 60 фото, собранными в 2008 году получилось 96 фотографий
2009 года. А в 2006 было собрано 60-36=24 фотографии. Обратите
внимание, как тонко здесь для задачи 3-4 классов вводятся свойства
последовательности Фибоначчи.
Ответ. Б: 24.
Задача 47. Какую наибольшую сумму очков мог получить Серёжа?
6*4=24. Полученная им сумма в 21 всего на 3 очка меньше максимальной.
А замена шестёрки тройкой также уменьшает максимальную сумму на 3.
Поэтому тройка могла выпасть всего раз.
Ответ. Б: 1.
Тема №10: целые числа, уравнения, логика
Условия задач
1.3.4-305
Задача 48. Сколько целых решений имеет уравнение
x(x+1)+(x+1)(x+2)+…+(x+9)(x+10)=1000x+1997?
А:0; Б:1; В:2; Г:6; Д: бесконечно много.
Задача 49. Число X состоит из цифр 1, 2, 3, а число Y – из цифр 4, 5,
6. Мы знаем, что число X+Y чётное и что вторая цифра числа X равна
двум. Какова последняя цифра числа X*Y?
А: нельзя однозначно установить; Б:2; В:6; Г:5; Д:4.
Задача 50. В тесте было 30 вопросов. Каждый правильный ответ
увеличивает количество набранных баллов на 7, а каждая ошибка или
отсутствие ответа уменьшает количество баллов на 12. Саша, выполнив
тест, набрал 77 баллов. Сколько ошибок он сделал?
А: от 0 до 4; Б: от 5 до 8; В: от 9 до 12; Г: о 13 до 16; Д: невозможно
определить.
Задача 51. Имеются 3 коробки и 3 предмета: монета, игрушечная
черепаха и горошина. У каждой коробке есть только один предмет,
причём:

Зелёная коробка находится левее голубой;

Монета находится левее горошины;

Красная коробка стоит правее черепахи;

Горошина правее красной коробки;
В какой коробке монета?
А: в красной; Б: в зелёной; В: в голубой; Г: невозможно определить
однозначно; Д: условия задачи противоречивы.
Задача 52. В обувном магазине для животных на 10 полках было по
12 пар обуви. Первыми покупателями были пять многоножек. Первые три
из них купили по 30 пар, а две следующие – по 5 пар каждая. Сколько пар
обуви осталось в магазине после визита этих покупателей?
А:10; Б:15; В:20; Г:25; Д:30.
1.3.4-306
Решения
Задача 48. Слева имеем сумму девяти произведений соседних целых
чисел. Из двух ярдом стоящих чисел одно всегда будет чётным, значит,
каждое из слагаемых в левой части – чётное число. И вся левая часть
уравнения будет чётной. Справа же стоит сумма чётного числа 1000х и
нечётного числа 1997. Значит, если бы некоторое целое х было бы
решением уравнения, то чётное число равнялось бы, нечётному, что
невозможно. Выходит, целых корней у уравнения нет.
Ответ. А: 0.
Задача 49. Поскольку двойка стоит внутри числа Х, то число Х
оканчивается на 1 или на 3, т.е. оно нечётное. Если сумма Х и Y – чётное
число, то число Y также нечётное. Значит, Y оканчивается на цифру 5.
Следовательно, произведение чисел Х и Y будет нечётным делящимся на 5
числом, и также будет оканчиваться на 5.
Ответ. Г: 5.
Задача 50. Пусть он решил x задач. Тогда 30-x задач остались без
правильного ответа. Общая сумма баллов составит 7x-12(30-x), что по
условию равняется 77. Решаем уравнение:
7x-12(30-x)=77
19x=437
x=23
Значит 7 вопросов остались без правильного ответа. Однако мы не
можем установить, были ли на них даны неверные ответы, или вообще
никаких ответов не давалось. Некоторая некорректность условия, которое
позволяло толковать отсутствие ответа на вопрос или как неправильный
ответ, или как отдельное действие, была исправлена тем, что при проверке
засчитывались два ответа:
Ответы. Б: от 5 до 8 или Д: невозможно определить.
1.3.4-307
Задача 51. Первое, на что обращаем внимание – это тройка
«черепаха – красная коробка – горошина». Её можно расположить лишь
одним способом:
Красная
Черепаха
Горошина
Единственное место для монеты – в красной коробке. Если бы среди
вариантов ответ не было намёка на возможность противоречивости
условия, на этом можно было бы остановиться. Однако нужно проверить,
возможна ли вообще ситуация, описанная в условии.
То, что зелёная коробка находится левее голубой, указывает нам цвета
коробок с черепахой и горошиной. А то, что монета находится левее
горошины, совпадает с уже найденным нами расположением.
Зелёная
Красная
Голубая
Черепаха
Монета
Горошина
Ответ. А: монета в красной коробке.
Задача 52. Решим задачу по вопросам
Вопрос 1. Сколько пар обуви было в магазине сначала?
10x12=120 (пар)
Вопрос 2. Сколько пар обуви купили первые три многоножки?
3x30=90 (пар)
Вопрос 3. Сколько пар обуви купили первые три многоножки?
2x5=10 (пар)
Вопрос 4. Сколько пар обуви было куплено всего?
10+90=100 (пар)
Вопрос 5. Сколько пар обуви осталось в магазине?
120-100=20 (пар)
Ответ. В: 20.
1.3.4-308
Тема №11: целые числа, делимость, палиндромы
Условия задач
Задача 53. Числами палиндромами называются такие числа, которые
читаются одинаково слева направо и справа налево. Сколько существует
пятизначных палиндромов, делящихся на 9?
А:81; Б:90; В:100; Г:500; Д:1000.
Задача 54. Каково наибольшее количество последовательных чисел,
ни у одного из которых сумма цифр не делится на 5?
А:5; Б:6; В:7; Г:8; Д:9.
Задача 55. Число 2004 делится на 12, а сумма его цифр равна 6.
Сколько четырёхзначных чисел имеют те же свойства?
А:10; Б:12; В:13; Г:15; Д:18.
Задача 56. 9 пирожных стоят меньше, чем 10 гривен, а 10 таких же
пирожных стоят больше, чем 11 гривен. Сколько стоит одно пирожное?
А: 1,09 грн.; Б: 1,11 грн.; В: 1,12 грн.; Г: 1,15 грн.; Д: невозможно
определить.
Задача 57. Между числами 2002 ? 2003 ? 2004 ? 2005 ? 2006 вместо
каждого знака вопроса можно записать знак + или -. Какое из чисел не
может получиться?
А:1988; Б:2001; В:2002; Г:2004; Д:2006.
Решения
Задача 53. Первую цифру числа мы можем выбрать 9-ю способами
(она может быть любой от 1 до 9). Последняя цифра определяется
однозначно. Вторую цифру можно выбрать 10-ю способами (она может
быть любой от 0 до 9), а предпоследняя определяется однозначно.
Сумма первой, второй, последней и предпоследней цифр числа
может давать остаток от 0 до 8 при делении на 9. Если данный остаток не
1.3.4-309
равен нулю, то средняя цифра определится однозначно как разность между
девятью и текущим остатком. А если же остаток равен нулю, то средняя
цифра может быть или нулём, или девяткой.
Поскольку числа 2 и 9 взаимно просты, то сумма двух первых и двух
последних цифр палиндрома будет делиться на 9 тогда и только тогда,
когда двузначное число, образованное первыми двумя его цифрами
делится на 9. И не будет делиться – в обратном случае.
Всего двузначных чисел – девяносто. Из них на 9 делятся 9 чисел:
18, 27, …, 90. Значит искомое число пятизначных палиндромов, делящихся
на 9 можно вычислить как 9х2+81=100.
Заметим, что этот ответ можно в тесте получить и намного быстрее.
Мы видим, что искомое число несколько больше количества способов
выбрать первые и последние 2 цифры числа, равного 9х10=90. А серди
вариантов ответа число 100 достаточно близко к 90, в то же время как
остальные варианты: 500 и 1000 уже отстоят от него довольно далеко. Так
что можно отвечать 100 и экономить время для других заданий.
Ответ. В: 100.
Задача 54. Обозначим сумму цифр числа n как S(n). Если n
оканчивается не на девятку, то S(n+1)= S(n)+1. так что если в
последовательности не будет «перехода через девятку», максимальная её
длина равна 4. Максимальная длина последовательности с «переходом
через девятку» будет, если сумма цифр первого после перехода числа
будет равна 1. Такое будет возможно, к примеру, для чисел 6, 7, 8, 9, 10,
11, 12, 13. Может ли быть 9 таких последовательных чисел? Нет. Ведь
среди девяти последовательных чисел может быть только один переход
через девятку, значит, или до, или после перехода будет идти 5
последовательных чисел. А среди них непременно найдётся число с
суммой цифр, делящейся на 5.
Ответ. Г: 8.
1.3.4-310
Задача 55. Если сумма цифр числа равна 6, оно автоматически будет
делиться на 3. Чтобы оно делилось, к тому же, на 4, двуциферное
окончание его должно делиться на 4. Поскольку сумма цифр в
двуциферном окончании должна быть меньше 6, то имеем следующие
варианты: 00, 20, 40, 12, 32, 04. Разберём, сколько будет способов
получения суммы первых двух цифр, дополняющей до 6 суму цифр
двуциферных окончаний.
00: нужно первыми двумя цифрами получить сумму 6. Всего 6 вариантов:
60, 51, 42, 33, 24, 15.
20: первыми двумя цифрами нужно получить сумму 4: всего 4 варианта:
40, 31, 22, 24.
40: сумма первых двух цифр должна быть 2. Имеем 2 варианта.
12: сумма первых двух цифр должна быть 3. Имеем 3 варианта.
32: для первых двух цифр остаётся единственный вариант.
04: для первых двух цифр есть 2 варианта.
Итого 6+4+2+3+1+2=18 вариантов.
Ответ. Д: 18.
Задача 56. Собственно, имея перед глазами варианты ответов, ответ
можно найти простым перебором. Но здесь мы разберём общий метод
решения подобных задач. Цена пирожного измеряется дискретной
величиной: гривнами и копейками. Обозначим её через натуральное х
число копеек. Имеем:
Единственное натуральное число, входящее в этот промежуток –
число 111. Поскольку это число копеек, то в гривнах цена составит 1,11.
Ответ. Б: 1,11.
1.3.4-311
Задача 57. Поскольку в полученном выражении будет два нечётных
числа, независимо от расстановки знаков, его значение будет чётным.
Значит, не может получиться число 2001.
Ответ. Б: 2001.
Тема №12: последовательности, проценты, уравнения
Условия задач
Задача 58. Выбранное число удваивают и отнимают единицу.
Повторяя эту процедуру 98 раз получили число 2^100+1. С какого числа
начинали?
А:1; Б:2; В:4; Г:6; Д: другое число.
Задача 59. За столом сидят 5 мальчиков и 6 девочек, а на столе на
тарелке лежат пирожки. Каждая девочка дала по одному пирожку каждому
знакомому мальчику. Потом каждый мальчик дал с тарелки по одному
пирожку каждой незнакомой девочке. После этого тарелка осталась
пустой. Сколько было пирожков на тарелке вначале?
А:5; Б:11; В:25; Г:30; Д: нельзя установить.
Задача 60. Некоторые из 11 коробок содержат по 8 меньших
коробок. Некоторые из меньших коробок содержат ещё но 8 коробок
каждая. Если пустых коробок 102, то сколько коробок всего?
А: 102; Б: 64; В: 118; Г: 115; Д: невозможно определить.
Задача 61. Если К составляет 10% от L, L составляет 20% от M, M
составляет 30% от N, P составляет 40% от N, то отношение К/Р равно:
А: 7; Б: 3/2; В: 2/300; Г: 3/200; Д: 1/250.
Задача 62. В соревнованиях по бегу участвовали 28 детей.
Количество детей, которые прибежали позже Димы вдвое больше
количества детей, которые прибежали раньше Димы. В таком случае Дима
прибежал:
1.3.4-312
А: шестым; Б: седьмым; В: восьмым; Г: девятым; Д: десятым.
Решения
Задача 58. Пусть начальное число равно а. Тогда мы будем получать
числа
2a-1
2(2a-1)-1=4a-3
2(4a-3)-1=8a-7
Можно заметить, что n-й результат выражается формулой 2^n(a-1)+1.
Докажем это по индукции. Для n=1 формула верна, пусть она верная для
n=k. Тогда для n=k+1: 2(2^k(a-1)+1)-1=2^k+1(a-1)+1. Сходится.
Теперь имеем:
2^98(a-1)+1=2^100+1
2^98(a-1)= 2^100
a-1=4
a=5
Значит, начинали с пятёрки.
Ответ. Д: с другого числа.
Задача 59.
Начертим граф знакомств между мальчиками и
девочками. Для этого нарисуем группы по 5 и 6
точек и соединим точи из разных групп красными
линиями, если соответствующие люди знакомы и
синими – если незнакомы.
Теперь
видно,
что
количество
пирожков,
переданных
девочками
мальчикам, равно количеству красных рёбер, а количество пирожков,
переданных мальчиками девочкам, равно количеству синих рёбер. Общее
количество рёбер равно 5х6=30. Вот столько и было пирожков на тарелке.
Такой граф, кстати, называется двудольным.
1.3.4-313
Ответ. Г: 30.
Задача 60. Пусть из 11 коробок «верхнего уровня» х заполнены
меньшим коробками. Тогда меньших коробок будет 8х. Пусть из них у
коробок заполнены маленькими. Тогда маленьких коробок будет 8у. Всего
коробок
будет
11+8х+8у=11+8(х+у).
Количество
пустых
коробок
подсчитаем так: пустых больших коробок будет 11-х, пустых меньших
коробок: 8х-у и все 8у маленьких коробок также будут пустыми. Всего
пустых коробок 11-х+8х-у+8у=11+7(х+у).
Из уравнения 11+7(х+у)=102 найдём сумму x+y
7(х+у)=91
x+y=13
Тогда всего коробок будет 11+8(х+у)= 11+7(х+у)+(х+у)=102+13=115.
Ответ. Г: 115.
Задача 61.
Отношение
Отношение
Искомое отношение находится как частное:
Ответ. Г: 3/200.
Задача 62. Остальных участников забега было 27. Если разделить 27
в отношении 1:2, получим 9 и 18. Значит, 9 участников финишировали
раньше Димы. Выходит, он пришёл десятым.
Ответ. Д: десятым.
Тема №13: принцип Дирихле, цифры, множества
Условия задач
1.3.4-314
Задача 63. Экипаж космического корабля, приземлившегося на
Марсе, заметил интересные особенности марсиан:

Все они или красные, или зелёные, или синие;

Рост каждого – 1 метр;

У марсианина от 2 до 5 голов;

На теле у них от 3 до 20 антенн.
Какое минимальное количество жителей должно быть в марсианском
посёлке, чтобы среди них заведомо можно было выбрать команду из 11
одинаковых игроков для футбольного матча с космонавтами? (Все 11
марсиан должны быть одного цвета, иметь одинаковое количество голов и
одинаковое количество антенн.)
А:216; Б:2161; В:2160001; Г:230051; Д: другое.
Задача 64. Пусть а=19971998+19981999+19992000+20002001. Чему равна
последняя цифра числа а?
А:0; Б:2; В:3; Г:4; Д:5.
Задача 65. В июне во Львове число солнечных дней составило 25%
от количества пасмурных, количество тёплых – 20% от количества
холодных. Только три дня были солнечными и тёплыми. Сколько было
пасмурных и холодных дней? (Всего в июне 30 дней.)
А: 27; Б: 22; В: 19; Г: 17; Д: 7.
Задача 66. У Полы и Билла вместе 18 гривен, у Билла и Джона – 12
гривен. У Джона и Марии – 10 гривен. Сколько гривен у Марии и Полы?
А: 16; Б: 20; В: 24; Г: 25; Д: 48.
Задача 67. Рассмотрим число 12321232123212321…, состоящее из
2002 цифр. Тремя последними цифрами этого числа будут:
А: 123; Б: 232; В: 321; Г: 212; Д: 321.
Решения
1.3.4-315
Задача 63. Рассмотрим, сколько всего может быть вариаций марсиан.
Имеем 3 возможности для цвета, 4 возможности для числа голов и 18
возможностей для количества антенн. Это даёт нам 3x4x18=216 вариантов.
Рассмотрим теперь самый неблагоприятный случай: в посёлке живёт по 10
марсиан каждой вариации. Тогда его население составит 2160 жителей.
Если же там поселится ещё один марсианин, он наверняка по своим
параметрам будет совпадать с некоторыми десятью жителями, формируя
тем самым местную футбольную команду.
Верно и обратное: имея 2161=216х10+1 жителей и 216 категорий,
согласно принципу Дирихле, как минимум в одной категории будет 11
представителей.
Ответ. Б: 2161.
Задача 64. Последние цифры степеней подчиняются несложным
закономерностям. Чтобы их вывести, достаточно понять, что последняя
цифра степени зависит только от последних цифр предыдущей степени и
основания.
Для семёрки получаем такую таблицу:
Показатель
1
2
3
4
5
Последняя
7
9
3
1
7
цифра
Заполняется она так: 71=7, 7х7=49. Во вторую ячейку пишем 9.
9х7=63 – пишем тройку в третью ячейку. 3х7=21 – пишем единицу. Далее
1х7=7 и цифры начнут повторяться.
Поскольку цифры повторяются через 4, а 1998 даёт остаток 2 при
делении на 4, то последней цифрой 19971998 будет 9.
Построим такие таблицы для восьмёрки и для нуля:
Показатель
1
2
3
4
5
1.3.4-316
Последняя
8
4
2
6
8
цифра
Отсюда, 19981999 оканчивается на 2.
Показатель
1
2
3
Последняя
9
1
9
цифра
А 19992000 – на 1.
Четвёртое
слагаемое,
понятно,
оканчивается
нулём.
Итого
9+2+1+0=12, значит, последней цифрой будет 2.
Ответ. Б: 2.
Задача 65. Поскольку солнечные дни относятся к пасмурным как 1:4,
то солнечных было 30/5=6, а пасмурных – 24. Т.к. тёплые дни относятся к
холодным как 1:5, то тёплых было 30/6=5, а холодных – 25.
Известно, что тёплых и солнечных дней было 3. Значит, 6-3=3
солнечных дня были холодными, а 5-3=2 тёплых дня было пасмурными.
Чтобы найти число холодных пасмурных дней, отнимем от общего
количества дней (30) тёплые солнечные (3), холодные солнечные (3) и
тёплые пасмурные (2). Итого 30-3-3-2=22 холодных пасмурных дня.
Ответ. Б: 22.
Задача 66. Чтобы найти сумму денег у Марии и Полы, сложим
деньги Полы и Билла с деньгами Джона и Марии, и отнять от результата
деньги Билла и Джона.
Получаем: 18+10-12=16 (грн).
Ответ. А: 16.
Задача 67. Можно заметить, что в заданном числе будет повторяться
группа цифра 1232. Т.к. общее количество цифр, 2002, даёт остаток 2 при
1.3.4-317
делении на 4, то число будет оканчиваться на …123212 и последними
тремя цифрами будут 212.
Ответ. Г: 212.
Тема №14: комбинаторика, делимость, геометрия
Условия задач
Задача 68. Тест состоит из 10 вопросов, на каждый из которых
нужно выбрать вариант ответа а) или б). Если на любые 5 вопросов
ответить вариантом а), а на остальные пять – вариантом б), то обязательно
как минимум 4 ответа окажутся верными. Сколько существуем вариантов
расположения правильных ответов в тесте, которые обеспечивают такое
его свойство?
А:2; Б:10; В:22; Г:252; Д: 5^5.
Задача 69. В коробке была 31 конфета. В первый день Кристина
съела 3/4 от количества конфет, которые съел Петя в тот же день. На
второй день Кристина съела 2/3 количества конфет, которые съел Петя в
тот же день. После двух дней коробка осталась пустой. Сколько конфет из
коробки съела Кристина?
А:9; Б:10; В:12; Г:13; Д:15.
Задача 70. Карл говорит правду в тот день, когда он не обманывает.
Какое из следующих утверждений Карл не мог высказать в один день
вместе с остальными?
А: Число моих друзей - простое;
Б: У меня столько же друзей среди мальчиков, сколько и среди девочек;
В: 288 делится на 12;
Г: Я всегда говорю правду;
Д: Три моих друга старше меня.
1.3.4-318
Задача 71. Какое наибольшее количество тупых углов могут
образовать 6 лучей с общим началом?
А: 6; Б: 8; В: 9; Г: 12; Д: 15.
Задача 72. В каждом подъезде на каждом этаже 16-этажного дома
есть по 4 квартиры. В каком подъезде и на каком этаже находится квартира
№165?
А: 3 подъезд 9 этаж; Б: 3 подъезд 10 этаж; В: 3 подъезд 12 этаж; Г: 2
подъезд 13 этаж; Д: 3 подъезд 7 этаж.
Решения
Задача 68. Если все правильные ответы будут а), то, отвечая
указанным в условии образом, мы будем иметь 5 гарантированных
правильных ответов. Рассмотрим теперь случай, если будет 9 правильных
ответов а) и один правильный ответ – б). Тогда в наихудшем случае все
наши ответы б) попадут на вопросы с ответом а), а из ответов а) один
попадёт на вопрос с правильным ответом б), а остальные 4 – на вопросы с
правильным ответом а). Так что 4 гарантированных правильных ответов
будет. Рассуждая далее аналогичным образом, получим, что в случае с
двумя правильными ответами б) и восемью – а), можно получить только 3
гарантированных верных ответа. Так что возможных распределений
правильных ответов в тесте, при котором описанный в условии трюк
сработает, всего 4: все ответы а), 9 а) и 1 б), а также, из соображений
симметрии, 1 ответ а) и 9 ответов б) и все ответы б).
Количество
возможных
вариантов
расположения
правильных
ответов в первом и четвёртом случаях будет по одному, а во втором и
третьем случаях – по 10. Итого 22 возможности.
Ответ. В: 22.
Задача 69. Из условия задачи следует, что количество конфет,
съеденных в первый день, делится на 7, а во второй – на 5. Представить
1.3.4-319
число 31 в виде суммы натуральных чисел, одно из которых делится на 7, а
другое – на 5, можно единственным способом: 31=21+10. Разбив первое
слагаемое в отношении 4:3, а второе – 3:2, получим 31=(12+9)+(6+4).
Значит, Кристина съела 9+4=13 конфет.
Ответ. Г:13.
Задача 70. Поскольку 288 на самом деле делится на 12, то
утверждение В – истинное. Из утверждений А, Б и Д следует, что число
его друзей простое, чётное и не меньше трёх. Очевидно, что никакое число
не может удовлетворять этим условиям, так что как минимум одно
утверждение из этой тройки – ложно. Следовательно, ложно и
утверждение Г о том, что он всегда говорит правду. Поскольку по условию
задачи, или среди 4 истинных утверждений будет одно ложное, или среди
4 ложных будет одно истинное, то но может быть сказано в один день с
остальными истинное утверждение о делимости.
Ответ. В: 288 делится на 12.
Задача 71. Всего 6 лучей сформируют 15 углов. Все 15 тупыми быть
не могут, потому что в таком случае мы не сможем уложиться в 360^o. А
вот 12 – вполне:
Ответ. Г: 12.
Задача 72. В одном подъезде 64 квартиры. Т.к. 165=2*64+9*4+1, то
квартира 165 будет в (2+1=3) третьем подъезде на (9+1=10) десятом этаже.
Ответ. Б: 3 подъезд 10 этаж.
1.3.4-320
Тема №15: системы счисления, геометрия, арифметика
Условия задач
Задача 73. Сколько положительных целых чисел могут быть
записаны как a0+a13+a232+a333+a434, если a0, a1, a2, a3, a4 принадлежат
множеству {-1, 0, 1}.
А:5; Б:80; В:81; Г:121; Д: 243.
Задача 74. Сколькими способами можно полностью покрыть
прямоугольник со сторонами 2x8 костяшками домино 1x2 без наложений?
А:16; Б:21; В:30; Г:32; Д:34.
Задача 75. По результатам контрольной работы, в классе средний
балл мальчиков оказался равен 8,6, девочек – 9,8, а средний балл всех
учеников в классе – 9,4. Какую часть класса составляют мальчики?
А: 1/4; Б: 1/3; В: 1/2; Г: 2/3; Д: невозможно определить.
Задача 76. Сколько точек пересечения точно не могут иметь 4
прямые?
А: 1; Б: 2; В: 3; Г: 4; Д: 5.
Задача 77. Маленький Мук и королевский скороход соревновались в
беге на дорожке длиной 30 км, которая проходила вокруг большого луга.
По условиям состязания, выиграет тот, кто обгонит другого, пробежав на
один круг больше. Скороход пробегает круг за 10 минут, а Маленький Мук
– за 6 минут. Оба стартуют одновременно из одного и того же места. Через
сколько минут Маленький Мук победит?
А: 5; Б: 10; В: 15; Г: 20; Д: 25.
Решения
Задача 73. Здесь мы имеем дело с равновесной троичной системой
счисления. Если бы коэффициенты при степенях тройки принимали
1.3.4-321
значения 0, 1 или 2, то таким образом могли бы быть записаны числа от 0
до (22222)3 = 242. А в равновесной троичной системе счисления пятью
цифрами можно записать число от ((-1)(-1)(-1)(-1)(-1))3 = -121 до (11111)3 =
121. Из них положительных чисел будет 121.
Строго доказать, что с помощью n цифр равновесной троичной
системы можно записать все числа в диапазоне от –(3n-1)/2 до (3n-1)/2
можно по индукции. Одной цифрой можно записать числа -1, 0, 1 – база
индукции имеется. Пусть для k цифр верно, что мы получаем все числа от
–(3k - 1)/2 до (3k - 1)/2.
Тогда, взяв k+1-ю цифру, равную нулю, мы получим те же числа.
Взяв её равной единице, получим числа от 3k – (3k - 1)/2 до 3k + (3k - 1)/2,
т.е. от (3k - 1)/2 +1 до (3k+1 - 1)/2. Аналогично, взяв взяв k+1-ю цифру,
равную -1, получим все числа от –(3k+1 - 1)/2 до (3k - 1)/2 -1. Таким образом,
с помощью k+1 цифры мы получим все числа от –(3k+1 - 1)/2 до (3k+1 - 1)/2,
что и требовалось доказать.
Следовательно, утверждение верно и с помощью 5 цифр мы
действительно получим все числа от -121 до 121.
Ответ. Г: 121.
Задача 74. У нас уже разбиралась задача о нахождении количества
разбиений полоски 2хn на домино 1х2. Там доказывалось, что искомое
количество равно n-му числу Фибоначчи, если начинать ряд с 1, 2.
Следовательно, для восьми, число способов будет равняться: 1, 2, 3, 5, 8,
13, 21, 34.
Ответ. Д:34.
Задача 75. Пусть доля мальчиков в классе равна x, 0<x<1. Тогда доля
девочек в классе (1-x). Верно равенство:
8,6x + 9,8(1-x) = 9,4.
8,6x + 9,8 - 9,8x = 9,4.
0,4 = 1,2x.
1.3.4-322
x = 1/3.
Значит, мальчиков в классе – треть.
Ответ. Б: 1/3.
Задача 76. Легко можно расположить 4 прямые так, чтобы у них
была одна или 4 точки пересечения. Если ещё немного подумать,
находятся варианты для трёх и пяти точек пересечения. Поскольку мы
имеем дело с тестом, теперь можно выбирать ответ Б: у четырёх прямых не
может быть ровно две точки пересечения.
Ответ. Б: 2.
Задача 77. Сколько километров пробегает скороход за минуту?
30/10 = 3 (км).
Сколько километров пробегает Маленький Мук за минуту?
30/6 = 5 (км).
На сколько
километров обгоняет Маленький
Мук обгоняет
скорохода каждую минуту?
5 – 3 = 2 (км).
Через сколько минут Маленький Мук обгонит скорохода на 30 км?
30/2 = 15 (мин).
Ответ. В: 15.
1.3.4-323
Тема №16: целые числа, логика, геометрия
Условия задач
Задача 78. Дядя Богдан наловил рыбы. Три самых больших рыбы он
дал своей собаке, тем самым, уменьшив общий вес своего улова на 35%.
Затем он дал три самых маленьких рыбы своему коту, уменьшив вес
оставшейся рыбы на 5/13. Остальные рыбы семья съела на обед. Сколько
рыб поймал дядя Богдан?
А:8; Б:9; В:10; Г:11; Д: 12.
Задача 79. В одной из подгрупп кубка чемпионов Европы
участвовали 5 команд:, A, B, C, D, E. Пять спортивных изданий высказали
свои прогнозы насчёт финалистов:
1)B, D;
2)C, E;
3)B, C;
4)A, B;
5)D, C.
Оказалось, что один из прогнозов был полностью верным, а в
остальных указывалась лишь одна из команд-финалистов. Какие команды
вышли в финал?
А: B, D; Б: C, E; В: B, C; Г: A, B; Д: D, C.
Задача 80. На плоскости даны 4 точки. Пять из шести расстояний
между ними равны 7, 5, 5, 2 и 2. Тогда шестое расстояние может равняться:
А: 3; Б: 4; В: 7; Г: 10; Д: 12.
Задача 81. Чтобы очистить 4 своих аквариума, Ваня поселил в них
улиток. Чтобы очистить один аквариум, нужны или 4 большие улитки, или
1 большая и 5 маленьких улиток, или 3 большие и 3 маленькие улитки. У
Вани 15 больших улиток. Но в зоомагазине он может обменять одну
1.3.4-324
большую улитку на 2 маленьких. Какое наименьшее количество больших
улиток нужно обменять Ване, чтобы почистить все свои аквариумы?
А: 2; Б: 3; В: 4; Г: 5; Д: 6.
Задача 82. В футбольном матче победитель получает 3 очка,
проигравший – 0, а ничья оценивается одним очком. После 31 матча моя
любимая команда имела 64 очка, причём 7 матчей она сыграла вничью.
Сколько раз проиграла моя любимая команда?
А: 0; Б: 5; В: 19; Г: 21; Д: 24.
Решения
Задача 78. Примем общий вес пойманных рыб за 100. Тогда вес
отданных коты рыб составил 35, после этого вес рыб составил 65. Коту
было отдано трёх рыб общим весом 25, следовательно, на обед осталось
несколько рыб весом 40.
Поскольку три самые тяжёлые рыбы имели общий вес 35, то вес
самой тяжёлой из оставшихся рыб не может быть меньше
вес самой лёгкой из оставшихся рыб не может быть меньше
. Аналогично,
.
Следовательно, количество оставшихся рыб находится между числами
и
. Единственное целое число в этом
диапазоне равно 4. Значит, всего рыб было 3+4+3=10.
Ответ. В: 10.
Задача 79. Перебором возможных вариантов находим такой прогноз,
который с каждым из остальных имеет один общий элемент. Это прогноз
третьего издательства: B, C.
Ответ. В: B, C.
Задача 80.
1.3.4-325
Как могут быть расположены те три из
данных
четырёх
точек,
расстояния
известны?
варианта:
в
между
Существует
вершинах
которыми
лишь
2
равнобедренного
треугольника с вершинами 7, 5, 5 или на отрезке
длиной 7. В первом случае установить четвёртую
точку так, чтобы использовать остальные 2 из известных расстояний,
невозможно. А второй вариант даёт нам отрезок длиной 7, разбитый на
отрезки в 2, 3 и 2. Следовательно, шестое расстояние равно 3.
Ответ. А: 3.
Задача 81. Относительно «обменного курса» улиток, второй вариант
очистки эквивалентен 3,5 большим улиткам, а третий способ – 4,5
большим. Поскольку у Вани всего 15=4+4+3,5+3,5 больших улиток, то ему
придётся 2 аквариума чистить первым способом, а ещё два – вторым. Для
этого ему нужно 5 больших улиток сменять на 10 маленьких.
Ответ. Г: 5.
Задача 82. За победы команда получила 64-7=57 очков. Значит,
побед было 57/3=19. Так как из 31 матча было 7 ничьих и 19 побед, то
поражений было 31-7-19=5.
Ответ. Б: 5.
Тема №17: тригонометрия, неравенства, геометрия
Условия задач
Задача 83. Каково максимальное значение выражения
sin a cos b + sin b cos c + sin c cos d + sin d cos a
для действительных a, b, c, d?
А:1; Б:2; В:3; Г:4; Д: 8.
1.3.4-326
Задача 84. Известно, что х и у - положительные действительные
числа, и только одно из приведённых в ответах утверждений истинное.
Какое?
А: x2 > 2y2; Б: x > 2y; В: x > y; Г: x2 > y2; Д: x > y2.
Задача 85. Некоторое количество прямых изобразили на бумаге так,
что между ними есть углы величиной 10°, 20°, 30°, 40°, 50°, 60°, 70°, 80°,
90°. Найдите наименьшее количество прямых, для которых такое
возможно.
А: 4; Б: 5; В: 6; Г: 7; Д: 8.
Задача 86. В стране Туфляндии у каждого жителя правая нога на
один или на два размера больше левой. К сожалению, в магазине
продаются пары обуви только одинакового размера. Чтобы сэкономить
деньги, несколько друзей пошли в магазин и каждый из них купил одну
пару обуви. Когда они обменялись обувью, один ботинок 36 размера и
один ботинок 45 размера оказались лишними. Какое наименьшее
количество человек могло быть в этой группе?
А: 5; Б: 6; В: 7; Г: 8; Д: 9.
Задача 87. На клумбе расцвели цветы: белый, красный, синий и
жёлтый. Пчела Майя подлетает к каждом цветку всего 1 раз. Сначала она
летит к красному цветку, а затем – к остальным. Майя не может лететь с
жёлтого цветка сразу на белый. Сколькими способами пчела Майя может
посетить все 4 цветка?
А: 1; Б: 2; В: 3; Г: 4; Д: 6.
Задача 88. Петя прибавляет 2, Назар отнимает 1, а Дима удваивает
число. Каждый мальчик выполняет своё действие только один раз. В каком
порядке им нужно выполнять эти действия, чтобы из 3 получить 9?
А: Дима, Петя, Назар;
Б: Петя, Дима, Назар;
В: Дима, Назар, Петя;
1.3.4-327
Г: Назар, Дима, Петя;
Д: Петя, Назар, Дима.
Решения
Задача 83. Выражение sin a cos b + sin b cos c + sin c cos d + sin d cos
a можно рассмотреть как скалярное произведение 4-мерных векторов с
координатами (sin a, sin b, sin c, sin d) и (cos b, cos c, cos d, cos a). По
неравенству Коши-Буняковского, скалярное произведение векторов не
превосходит произведения их модулей. Значит:
Применив теперь неравенство между средним геометрическим и средним
арифметическим, получим:
Взяв
получаем значение выражение, в точности
равное двум.
Ответ. Б:2.
Задача 84. Установим, какие из приведённых соотношений
взаимосвязаны.
А: x2 > 2y2; => Г: x2 > y2;
Б: x > 2y; => В: x > y;
В: x > y; <=> Г: x2 > y2.
Выходит, утверждения А, Б, В, Г не могут быть единственными
истинными, т.к. их истинность влечёт за собой истинность какого-нибудь
1.3.4-328
ещё утверждения. Значит, они ложны, а единственно истинным будет
утверждение x > y2. Примером таких чисел будут
.
Ответ. Д: x > y2.
Задача 85. Заметим, что среди прямых, две обязательно должны
быть перпендикулярны. Если две перпендикулярные прямые пересечь ещё
двумя, острых углов может получиться не более пяти, как в случае, когда
ни одна из этих двух прямых не проходит через точку пересечения
перпендикулярных (на рисунке), так и в других случаях.
Имея же 5 прямых, мы можем построить требуемую конструкцию:
Ответ. Б: 5.
Задача 86. Наименьшим количество покупателей будет, если у
наибольшего их количества ноги различаются на 2 размера. Значит, это
были люди с размерами: (45, 43), (43, 41), (41, 39), (39, 37) и (37, 36) –
итого 5 человек. Но ответить 5 было бы опрометчиво. Ведь кроме обутых в
1.3.4-329
итоге ботинок было куплено ещё 2 штуки. Значит, всего купили 6 пар
ботинок, и покупателей было шестеро.
Ответ. Б: 6.
Задача 87. Первый цветок она выбирает однозначно. Второй цветок
может быть выбран одним из трёх способов:
Красный – Белый, Красный – Синий или Красный – Жёлтый.
Поскольку с жёлтого цветка нельзя лететь сразу на белый, получаем 5
способов для трёх цветков:
Красный – Белый – Жёлтый,
Красный – Белый – Синий,
Красный – Синий – Белый,
Красный – Синий – Жёлтый,
Красный – Жёлтый – Синий.
Но среди этих способов один путь, а именно, Красный – Синий –
Жёлтый – тупиковый, так как никакой цветок, кроме белого, не остаётся, а
на него лететь нельзя. Остальные же 4 тройки дают нам 4 возможных
маршрута облёта цветов:
Красный – Белый – Жёлтый – Синий,
Красный – Белый – Синий – Жёлтый,
Красный – Синий – Белый – Жёлтый,
Красный – Жёлтый – Синий – Белый.
Ответ. Г: 4.
Задача 88. Поскольку 9=(3+2)*2-1, то сначала посчитать должен
Петя, затем Дима, и потом – Назар.
Ответ. Б: Петя, Дима, Назар.
1.3.4-330
Тема №18: тригонометрия, последовательность, цифры
Условия задач
Задача 89. Найдите, при каких значениях острого угла a уравнение
(2cosa -1)x2 - 4x + 4cosa + 2 = 0
будет иметь два действительных положительных корня?
А:0o < a < 30o; Б: 0o < a < 60^0; В: 30o < a < 60^0; Г: 30o < a < 90^0; Д: 0o < a
< 90o.
Задача 90. Последовательность целых чисел задаётся рекуррентно:
a0=1, a2=2, an+2=an+(an+1)2. Чему равен остаток от деления a2009 на 7?
А: 0; Б: 1; В: 2; Г: 5; Д: 6.
Задача 91. Решением уравнения (x+22007)2 – (x–22007)2 = 22008 является:
А: 0,5; Б: 2; В: 22; Г: 22008; Д: 0.
Задача 92. Комплект домино состоит из 28 костяшек, которые
образованы всеми возможными комбинациями количеств точек от 0 до 6
включительно. Сколько всего точек в наборе домино?
А: 84; Б: 105; В: 126; Г: 147; Д: 168.
Задача 93. Сколько существует двузначных чисел, у которых цифра
справа больше цифры слева?
А: 9; Б: 18; В: 26; Г: 30; Д: 36.
Задача 94. Секретный агент хочет расшифровать код из шести цифр.
Он знает, что сумма цифр на первом, третьем и пятом местах равна сумме
цифр на втором, четвёртом и шестом местах. Какой из предложенных
вариантов не может быть кодом?
А: 81**61;
Б: 7*727*;
В: 4*4141;
Г: 12*9*8;
Д: 181*2*.
1.3.4-331
Решения
Задача
89.
Раз
корни
этого
уравнения
положительны,
положительной будет и их сумма, которая по теореме Виета равняется:
Это исключает ответы Г и Д.
Дискриминант должен быть положительным, значит
D=16-8(2 cosa-1)(2 cosa+1)=16-8(4 cos2 a-1)=24-32 cos2 a >0.
Отсюда,
Отбрасываем ответы А и Б, оставляя единственный вариант.
Ответ. В: 30o < a < 60o.
Задача 90. Сначала найдём a1 из уравнения: 2=1+ (a1)2, откуда a1=1
или a1=-1.
Для каждого варианта далее будем вместо самих an вычислять
остатки от деления an на 7.
При a1=1 получим последовательность:
1, 1, 2, 5, 6, 6, 0, 6, 1, 0, 1, 1, ...
Поскольку десятый и одиннадцатый члены равны единице и каждый
член последовательности однозначно определяется двумя предыдущими,
то далее последовательность зациклится циклом (1,1,2,5,6,6,0,6,1,0).
Период цикла равен 10. Значит a2009 будет давать такой же остаток при
делении на 7, как и a9, а именно 0.
При a1=-1 последовательность остатков будет такой же, если бы a1
равнялось 6: 1, 6, 2, 3, 4, 5, 1, 6,
Здесь длина периода равна 6, и т.к. 2009 даёт остаток 5 при делении
на 6, то a2009=a5=5.
1.3.4-332
Ответ. А: 0 или Г: 5.
Задача 91. Раскроем скобки:
(x+22007)2 – (x–22007)2 = 22008
x2+22008x+24014 - x2+22008x-24014 = 22008
22009x = 22008
x=0,5
Ответ. А: 0,5.
Задача 92. В наборе будет 7 дублей, костяшек с одинаковыми
значениями на половинках. Сумма очков на всех дублях равна
(0+1+2+3+4+5+6)х2=21х2=42. В остальном наборе каждое количество
точек на костяшках домино появляется в паре с шестью остальными.
Например: 3-0, 3-1, 3-2, 3-4, 3-5, 3-6. так что нужно найти значение
выражения (0+1+2+3+4+5+6)х6=21х6=126. Но т.к. при таком подсчёте
каждая костяшка была учтена дважды (например, как 3-4 и как 4-3), то
результат нужно разделить на 2: 126/2=63. Вместе с дублями количество
очков составит 42+63=105.
Ответ. Б: 105.
Задача 93. Среди чисел, которые начинаются на 1, таких чисел будет
8: от 12 до 19. среди начинающихся на 2 их будет 7: от 23 до 29. И т.д., для
начинающихся на 8 будет всего одно число – 89, а для следующего десятка
таких не будет совсем. Ответом будет сумма 8+7+6+5+4+3+2+1=4х9=36.
Ответ. Д: 36.
Задача 94. Рассмотрим для каждого из вариантов, может ли
выполниться условие:
А: 81**61;
8+*+6=1+*+1
14+*=2+*
12+*=*
1.3.4-333
На местах звёздочек должны стоять цифры, различающиеся на 12,
что невозможно.
Б: 7*727*;
7+7+7=*+2+*;
19=*+*.
Две цифры не могут дать в сумме 19.
В: 4*4141;
4+4+4=*+1+1;
10=*.
Невозможно.
Г: 12*9*8;
1+*+*=2+9+8;
*+*=18.
А вот это возможно, т.к. 9+9=18 и кодом будет последовательность 129998.
Д: 181*2*;
1+1+2=8+*+*.
Здесь правая часть уже явно больше левой.
Так что единственный вариант ответа, который может быть кодом - это
12*9*8.
Ответ. Г: 12*9*8.
Тема №19: комбинаторика, последовательность, логика
Условия задач
Задача 95. Ордината вершины параболы y=x2+bx+c равна -7.
Сколько целых чисел может находиться между корнями уравнения
x2+bx+с=0?
А:6 или 7; Б: 4 или 5; В: 5 или 6; Г: только 5; Д: только 6.
1.3.4-334
Задача 96. Кенгуру прыгает только вперёд на 1 или на 3 метра. Он
хочет преодолеть ровно 10 метров. Сколькими способами он может это
сделать?
А: 28; Б: 34; В: 35; Г: 55; Д: 56.
Задача 97. Дана числовая последовательность такая, что a1=1, a2=2,
a3=3, an+3= an+ an+1– an+2. Найдите a2007.
А: -2006; Б: -2004; В: -2002; Г: 2008; Д: 2007.
Задача 98. Пять целых чисел написали по кругу так, что сумма
никаких двух или трёх расположенных подряд не делится на 3. Сколько
среди этих пяти чисел таких, которые делятся на 3?
А: 0; Б: 1; В: 2; Г: 3; Д: невозможно определить.
Задача 99. Есть 5 коробок с карточками с буквами B, R, A, V, O.
В первой лежат B, V.
Во второй лежат B, A, V, R.
В третьей лежат A, B.
В четвёртой лежит V.
В пятой лежат B, R, A, V, O.
Петя вытащил из коробок карточки так, чтобы в каждой коробке
осталось по одной карточке и в разных коробках остались карточки с
разными буквами. Какая буква останется во второй коробке?
А: B; Б: R; В: A; Г: V; Д: O.
Задача 100. Маша подарила маме, бабушке, тёте и двум сёстрам по
букету цветов. Цветы для сестёр и тёти были одного цвета. Известно, что
бабушке она подарила не розы. Какой из этих букетов получила мама?
А: Жёлтые тюльпаны; Б: Розовые розы; В: Красные гвоздики; Г: Жёлтые
розы; Д: Жёлтые гвоздики.
1.3.4-335
Решения
Задача 95. Рассмотрим параболу y=x2–7. Точки пересечения её с
осью Ох имеют абсциссы
равно
, следовательно, расстояние между ними
. При сдвиге параболы вдоль оси Ох, расстояние между
корнями не поменяется, и т.к.
, между корнями будут попадать
5 или 6 целых чисел.
Ответ. В: 5 или 6.
Задача 96. Есть 4 варианта представления числа 10 в виде суммы
троек или единиц:
10=1+1+1+1+1+1+1+1+1+1=1+1+1+1+1+1+1+3=1+1+1+1+3+3=1+3+3+3.
Первое разбиение предлагает всего один способ преодоления
расстояния в 10 метров.
Во втором разбиении единственная тройка может быть одним из
восьми слагаемых, что даёт 8 вариантов.
Из
третьего
разбиения
можно
получить
вариантов
расположения прыжков в 3 метра.
В третьем разбиении единственная единица может быть одним из
четырёх слагаемых, что даёт ещё 4 варианта.
Всего 1+8+15+4=28 вариантов.
Ответ. А: 28.
Задача 97. Вычислим несколько членов последовательности: 1, 2, 3,
0, 5, -2, 7, -4, 9, -6, 11, -8, 13, -10, 15, -12, 17.
Замечаем, что члены с нечётными номерами равны номеру. А члены
с чётными номерами равны разности между числом 2 и номером.
a2n+1=2n+1
a2n=2-2n
Докажем это по индукции. Базу мы уже построили. Пусть это
условие выполняется для всех чисел с номером, меньшим 2k. Тогда
1.3.4-336
a2k= a2k-3+ a2k-2– a2k-1 = 2k-3 + 2-(2k-2) – (2k-1) = 2-2k
a2k+1= a2k-2+ a2k-1– a2k = 2-(2k-2) + (2k-1) – (2-2k) = 2k+1
Доказано.
Следовательно, a2007 = 2007.
Ответ. Д: 2007.
Задача 98.
Рассмотрим остатки от деления записанных
чисел на 3. Могут ли три из них быть равными 0?
Нет, т.к. в таком случае 2 числа стояли бы рядом, и
их сумма делилась бы на 3.
Что если два из остатков равняться 0? Да, но в
таком случае между ними должен стоять некоторый
нулевой остаток, скажем, 1. Пусть числа А и С
делятся на 3, а В даёт остаток 1. Тогда остатки E и
D должны равняться только единицам, иначе три
рядом стоящих числа разделятся на 3. Получаем
удовлетворяющее условию расположение.
Может ли только один из остатков равняться
0? Пусть А даёт остаток 0. Тогда у В и Е должны
быть одинаковые ненулевые остатки, иначе или
сумма одной из пар, или всех трёх чисел разделится на 3. Допустим, они
равны 1.
Следовательно, ни один из остатков С и D не равен 2. Также они не
могут одновременно равняться 1. Значит, один из них равен 0, а другой – 1.
Но этот случай с двумя числами, делящимися на 2, мы уже рассмотрели.
Может ли ни одно число не делиться на 3? Нет, т.к. в таком случае
найдётся три подряд стоящих одинаковых остатка, в сумме дающих
делящееся на 3 число.
Следовательно, ровно 2 числа из пяти должны делиться на 3.
1.3.4-337
Ответ. В: 2.
Задача 99. Из четвёртой коробки ничего не нужно вытаскивать. Там
останется V. Значит, из первой нужно вытащить V и оставить В.
Тогда из третьей нужно вытащить В и оставить А. И из второй нужно
вытащить B, A, V и оставить R. Из пятой тогда Петя вытащит всё, кроме О.
Ответ. Б: R.
Задача 100. Три жёлтых букета: А, Г и Д получили тётя и сёстры.
Т.к. бабушка получила не розы, то она получила гвоздики, а маме Маша
подарила розовые розы.
Ответ. Б: Розовые розы.
1.3.4-338
ТЕСТОВЫЕ ЗАДАНИЯ ПО ФИЗИКЕ
Раздел механика
1. Шарик массой m = 100 г подвешен на нити длиной L = 50 см.
Нить отклонили на угол  = 600 от вертикали и отпустили. Нить
обрывается, когда скорость шарика достигает значения V = 2 м/с
Определить силу натяжения нити в момент разрыва.
○ 3,38
◙ 1,69 Н
○ 2,63
○ 16,9
2. Лента горизонтального транспортера движется со скоростью U = 1
м/с. На ленту по касательной к ее поверхности влетает шайба, начальная
скорость которой V = 2 м/с перпендикулярна краю ленты. Найти
максимальную ширину ленты, при которой шайба достигнет ее
противоположного края, если коэффициент трения между шайбой и
лентой равен 0,2.
○ 2,28
◙ 1,14
○ 0,57
○ 11,4
3. При равноускоренном движении из состояния покоя перемещение
тела за пятую секунду после начала движения S = 10 м. Найти
перемещение тела за седьмую секунду.
○ 7,2 м
1.3.4-339
○ 72 м
◙ 14,4 м
○ 28,8 м
4. Длина недеформированной пружины L = 10 см и возрастает в 1,2
раза, если в вертикальном положении к ней подвесить груз А. Один конец
пружины закрепили на гладком столе, а груз А заставили скользить по
столу под действием силы упругости пружины по окружности со
скоростью V = 1 м/с. Найти радиус окружности.
○ 23,4 см
○ 2,34 м
◙ 11,7 см
○ 11,7 м
5. На вершине гладкой полусферы радиуса R = 0,7 м находится
шайба массой m = 10 г. Шайба начала скользить вдоль сферы под
действием горизонтально направленного
кратковременного импульса
силы, равного 0,03 Н.с. На какой высоте от основания полусферы шайба
оторвется от ее поверхности?
◙ 0,6 м
○ 1,2 м
○ 2,4 м
○ 0,3 м
6. Огнетушитель выбрасывает ежесекундно в горизонтальном
направлении m = 200 г пены со скоростью V = 20 м/с. Вес полного
огнетушителя равен 20 Н. Какую силу должен приложить человек,
чтобы удержать огнетушитель неподвижным в начальный момент его
работы?
1.3.4-340
○ 10,2 Н
○ 80 Н
○ 200 Н
◙ 20,4 Н
7. От поезда, идущего по горизонтальному участку пути с
постоянной скоростью V = 15 м/с, отцепляется 1/3 часть состава. Через
некоторое время скорость отцепившихся вагонов уменьшилась в два раза.
Считая, что сила тяги при разрыве состава не изменилась, определить
скорость головной части поезда в этот момент. Считать силу трения
пропорциональной весу тела.
○ 38 м/с
◙ 19 м/с
○ 3,8 м/с
○ 187 м/с
8. Площадь сечения сопла авиационного двигателя равна 1 м2.
Плотность газа в струе  = 2 кг/м3. Определить силу тяги двигателя, если
скорость истечения газовой струи равна 200 м/с.
○ 40 кН
○ 20 кН
◙ 80 кН
○ 160 кН
9. Период обращения вокруг Солнца для Меркурия равен 0,241 года.
Рассчитать радиус орбиты этой планеты, если известны: радиус Солнца RС
= 700 тыс. км и ускорение свободного падения на поверхности Солнца gС
= 265 м/с2.
○ 326,5 млн. км
1.3.4-341
◙ 57,9 млн. км
○ 32 млн. км
○ 115 млн. км
10.
Две
звезды
под
действием
взаимного
гравитационного
притяжения вращаются вокруг общего центра с периодом T = 2 года.
Масса каждой звезды равна массе Солнца. Во сколько раз расстояние
между звездами больше среднего расстояния от Земли до Солнца?
○ 4
○ 12
◙ 2
○ 8
11. Показания динамометра, к которому подвешено тело, на экваторе
некоторой планеты на 10% меньше, чем на полюсе. Определить среднюю
плотность планеты, если продолжительность суток на ней равна 6 часам.
○ 100 т/м3
○ 600 т/м3
○ 30 т/м3
◙ 300 т/м3
12. Маленький шарик подвешен на нити и
отклонен от положения равновесия на угол  = 900.
L
Длина нити L = 80 см. На расстоянии l = 40 см под
точкой
подвеса
горизонтальная
нити
плита.
расположена
На
какую
стальная
l
высоту
поднимется шарик после абсолютно упругого удара
о плиту?
○ 100 см
1.3.4-342
◙ 30 см
○ 10 см
○ 20 см
13. Вентилятор гонит струю воздуха через отверстие в стене. Во
сколько раз необходимо увеличить мощность вентилятора, чтобы
перегоняемое в единицу времени количество воздуха увеличилось в два
раза? Плотность воздуха считать постоянной.
○ 3
○ 4
○ 7
◙ 8
14. Какая мощность затрачивается насосом для перекачки воды по
трубе на высоту H = 2м? Площадь сечения трубы S = 1,5 см2. Насос за t =
1 с перекачивает объем V = 0,4 л воды. Трением пренебречь.
◙ 9,26 Вт
○ 36,7 Вт
○ 28,4 Вт
○ 4,63 Вт
15. С помощью веревки тело равномерно
поднимают вверх по наклонной плоскости. Веревка
направлена вдоль наклонной плоскости. При каком
угле при основании  затраченная мощность будет
максимальна? Коэффициент трения между телом и
наклонной плоскостью k =
3.
3

○ 30º
1.3.4-343
○ 45º
○ 15º
◙ 60º
Раздел статика и гидростатика
16. В двух сообщающихся цилиндрических сосудах с одинаковыми
поперечными сечениями площадью S = 11,5 см2 находится ртуть. В один
из сосудов поверх ртути наливают M = 1 кг воды и опускают плавать тело,
масса которого m = 0,15 кг. На какую высоту поднимется уровень ртути во
втором сосуде? Плотность ртути рт=13600 кг/м3.
◙ 3,7 см
○ 37 см
○ 17 см
○ 2,4 см
Ответ: 3,7 см.
17. В цилиндрическую банку с водой опустили железную коробочку,
из-за чего уровень воды в банке повысился на h = 3 см. Насколько
опустится уровень воды, если коробочка утонет? Плотность железа ж=
7800 кг/м3, воды о=1000 кг/м3.
○ 1,5 см
○ 3 см
◙ 2,6 см
○ 3,9 см
18. Слиток сплава, состоящий из золота и серебра,
взвешивают в воздухе и в воде. Показания динамометра
1.3.4-344
соответственно равны 3 Н и 2,75 Н. Определить массу
золота в слитке, считая, что при изготовлении сплава
суммарный объем не изменяется. Плотность золота з=
19,3 г/см3, серебра с= 10,5 г/см3, воды о= 1г/см3.
◙ 84 г
○ 37 г
○ 96 г
○ 42 г
19. Шарик для игры в настольный теннис радиусом r = 15 мм и
массой m = 5 г погружают в воду на глубину H = 30 см. Когда шарик
отпустили, он выпрыгнул из воды на высоту h = 10 см. Какое количество
тепла при этом выделилось вследствие трения шарика о воду? Плотность
воды о=1000 кг/м3.
○ 96,4 мДж
○ 48,2 мДж
◙ 21,8 мДж
○ 10,4 мДж
20. Плоская льдина площадью S = 1 м2 и высотой H = 0,4 м плавает в
воде. Какую работу надо совершить, чтобы полностью погрузить льдину в
воду? Плотность льда равна 900 кг/м3, воды - 1000 кг/м3.
○ 15,2 Дж
◙ 7,84 Дж
○ 3, 56 Дж
○ 0,24 Дж
21. В трех одинаковых цилиндрических сообщающихся сосудах
находится ртуть. В первый сосуд налили слой воды высотой Н = 102 мм, а
1.3.4-345
во второй в 1,5 раза больше воды. Насколько повысится уровень ртути в
третьем сосуде? Плотность ртути рт = 13600 кг/м3, плотность воды о=
1000 кг/м3.
○ 3,14 мм
○ 1,17 мм
◙ 6,25 мм
○ 15,2 мм
23. В цилиндрическом сосуде высота налитой воды равна 30 см.
Когда в сосуд опустили пустой стеклянный стакан так, чтобы он плавал,
уровень воды в сосуде поднялся на h = 2,2 см. Чему будет равна высота
воды в сосуде, если стакан утопить в воде? Плотность стекла ст = 2700
кг/м3, воды о = 1000 кг/м3.
○ 15 см
◙ 31 см
○ 28 см
○ 7 см
24. С какой высоты должно падать небольшое тело, плотность
которого равна 0,4 г/см3, чтобы оно погрузилось в воду на глубину h = 6
см? Сопротивление воды и воздуха при движении тела не учитывать.
○ 15 см
◙ 9 см
○ 28 см
○ 7 см
25. В цилиндрическом стакане с водой плавает кубик с
длиной ребра а = 5 см. Площадь сечения стакана равна 50 см2.
1.3.4-346
Высота воды в стакане тоже равна 5 см. При помощи тонкой
спицы кубик медленно опускают на дно. Какая при этом
совершается работа? Плотность кубика равна 500 кг/м3, воды 1000 кг/м3.
◙ 11,5 мДж
○ 6,7 мДж
○ 23 мДж
○ 17,4 мДж
26. Два одинаковых шара каждый из которых имеет
радиус r = 4 см и массу m = 120 г положены в вертикальный
открытый с обеих сторон полый тонкостенный цилиндр
радиусом R = 6 см, стоящий на горизонтальной поверхности.
Какой должна быть минимальная масса полого цилиндра,
чтобы шары не могли его опрокинуть?
○ 40 г
◙ 80 г
○ 20 г
○ 60 г
27. К невесомой пружине, первоначальная длина которой l = 21 см,
подвешивают груз. При этом длина пружины увеличивается на  l = 0,1l. В
какой точке нерастянутой пружины необходимо подвесить тот же груз,
чтобы он оказался на одинаковом расстоянии от концов пружины?
○ в 5 см от верхнего края
◙ в 10 см от верхнего края
○ в 8 см от верхнего края
○ в 4 см от верхнего края
1.3.4-347
28. Прикрепленный к вертикальной пружине груз медленно
опускают до положения равновесия. При этом пружина растягивается на
длину l = 5 см. Какой максимальной скорости достигнет этот груз, если
ему предоставить возможность падать свободно с такого положения, при
котором пружина не растянута?
◙ 0,7 м/с
○ 0,2 м/с
○ 0,5 м/с
○ 0,4 м/с
Раздел колебания
29. Точные астрономические часы с секундным математическим
маятником установлены в подвале здания МГУ. Насколько будут отставать
за неделю эти часы, если их перенести на один из верхних этажей здания,
находящегося на h = 100 м выше подвального помещения? Радиус Земли
равен 6400 км.
○ 4,12 с
○ 1,35 с
◙ 9,45 с
○ 18,9 с
30. Горизонтальный пружинный маятник вывели из положения
равновесия и отпустили. Через какое время кинетическая энергия
колеблющегося тела сравняется с потенциальной энергией пружины, если
период колебаний маятника равен 0,8 с? Трением пренебречь.
◙ 0,1 с
○ 0,2 с
1.3.4-348
○ 0,3 с
○ 0,4 с
31. Подвешенный к пружине груз массой m = 0,2 кг совершает 30
колебаний за 1 мин с амплитудой A = 0,1 м. Определить кинетическую
энергию груза через 1/6 часть периода после момента прохождения грузом
положения равновесия.
○ 5,23 мДж
◙ 2,4 мДж
○ 4,8 мДж
○ 28 мДж
Молекулярная физика и термодинамика
31. В узкой цилиндрической запаянной с одного конца трубке
находится воздух, отделенный от наружного пространства столбиком
ртути длиной h = 15 см. Каково атмосферное давление, если трубка
вначале лежит горизонтально, а затем ставится вертикально и при этом
объем воздуха в ней уменьшается от 241 мм3 до 200 мм3. Температуру
считать постоянной. Плотность ртути равна 13600 кг/м3.
○ 27 кПа
◙ 97 кПа
○ 48 кПа
○ 115 кПа
32. На дне вертикального запаянного цилиндра лежит стальной
полый шарик массой m = 5 г, радиус которого r = 2 см. Цилиндр
заполняют через клапан воздухом, имеющим температуру Т = 293 К.
Каким должно стать давление воздуха в цилиндре, чтобы шарик начал
1.3.4-349
подниматься вверх? Молярная масса воздуха равна 29 кг/кмоль. Считать,
что воздух при заданных условиях подчиняется уравнению состояния
идеального газа.
◙ 12,5 МПа
○ 6,8 МПа
○ 24 МПа
○ 48 МПа
33. Идеальный газ расширяется так, что его давление меняется в
соответствии с уравнением Р = аV , где V- объем газа, коэффициент а
равен 10 МПа/м3. Найти графически работу, произведенную газом при
расширении от объема V1= 10 л до V2= 20 л.
○ 3000 Дж
○ 4500Дж
◙ 1500 Дж
○ 750 Дж
34. В цилиндре под невесомым поршнем находится газ под
давлением, равным внешнему давлению, и с температурой t = 10
0
С. Поршень соединен с дном сосуда пружиной. До какой
температуры
необходимо
нагреть
газ,
чтобы
его
объем
увеличился в 1,5 раза? Считать, что, если газ из-под поршня
полностью откачать, то поршень будет находиться в равновесии
около дна цилиндра, не касаясь его.
○ 158 ºС
○ 464 ºС
○ 237 ºС
1.3.4-350
◙ 364 ºС
35. Пузырек воздуха поднимается со дна озера к поверхности воды.
Какова глубина озера, если объем пузырька при подъеме увеличивается в
два раза? Атмосферное давление Р0 = 0,1 МПа, температуру воздуха
внутри пузырька считать постоянной, плотность воды равна 1000 кг/м3.
○ 8м
◙ 10 м
○ 14 м
○ 5м
36. Баллон емкостью V = 50 л наполнен воздухом при температуре t
= 27 ºС до давления P = 10 МПа. Какой объем воды можно вытеснить из
балластного резервуара подводной лодки воздухом этого баллона, если
вытеснение производить на глубине H = 40 м? Температура воздуха после
расширения в резервуаре равна 3 0С. Плотность воды равна 1000 кг/м3.
◙ 925 л
○ 174 л
○ 457 л
○ 1,540 л
37. В вертикальном цилиндре под тяжелым поршнем находится
кислород массой m = 2 кг. Для повышения температуры кислорода на T =
5 К ему было сообщено количество теплоты Q = 9160 Дж. Найти
изменение внутренней энергии кислорода. Молярная масса кислорода
равна 32 кг/кмоль.
○ 3,28 кДж
◙ 6,57 кДж
○ 17,3 кДж
1.3.4-351
○ 24,5 кДж
38. Кислород массой m = 20 г охлаждается изохорно от температуры
T = 600 К так, что его давление падает в 2 раза. Затем газ расширяется при
постоянном давлении. В конечном состоянии его температура равна
первоначальной. Определить совершенную при этом газом работу.
Молярная масса кислорода равна 32 кг/кмоль.
○ 3,28 кДж
◙ 1,55 кДж
○ 17,3 кДж
○ 24,5 кДж
Ответ: 1,55 кДж.
39. Объем воздушного шара равен 224 м3, масса его оболочки m =
145 кг. Шар наполнен горячим воздухом при атмосферном давлении P =
100 кПа. Какую температуру должен иметь воздух внутри оболочки, чтобы
шар начал подниматься? Температура воздуха вне оболочки равна 0 0С,
молярная масса воздуха M = 29 кг/кмоль.
○ 364 ºС
◙ 273 ºС
○ 164 ºС
○ 263 ºС
40. Три одинаковых сосуда, соединенные тонкими непроводящими
тепло трубками, заполнены при температуре 10 К некоторым количеством
гелия. Затем один сосуд нагрели до температуры 50 К, а другой до 100 К.
Температура третьего сосуда осталась неизменной. Во сколько раз
изменилось давление в системе? Объемом соединительных трубок
пренебречь.
◙ 2,3
1.3.4-352
○ 3,2
○ 1,7
○ 4,6
Раздел электричество и магнетизм
41. Электродвигатель постоянного тока, имеющий сопротивление
обмотки R = 1 Ом, включен в сеть с напряжением U = 10 В. Какую
максимальную мощность может развить двигатель?
○ 36 Вт
○ 12 Вт
◙ 25 Вт
○ 47 Вт
42. Конденсатор, присоединенный к батарее с ЭДС равной 2 кВ
проводами с сопротивлением r = 0,1 Ом, имеет емкость C1= 2 мкФ. С
некоторого момента в течение интервала времени t = 10 с его емкость
равномерно увеличивается до C2= 7 мкФ. Какая тепловая энергия
выделится при этом в подводящих проводах?
◙ 2 мкДж
○ 1 мкДж
○ 4 мДж
○ 5 мДж
43. К источнику постоянного тока с ЭДС равной 12 В и внутренним
сопротивлением r = 2 Ом подключен реостат. Какова максимально
возможная мощность тока в реостате.
◙ 18 Вт
○ 9 Вт
1.3.4-353
○ 12 Вт
○ 24 Вт
44. Конденсатор емкостью C = 5 мкФ и резистор с сопротивлением R
= 12 Ом соединены последовательно. Эта цепь подключают к источнику
постоянного
тока,
ЭДС
которого
равна
1000
В,
а
внутреннее
сопротивление r = 3 Ом. Какое количество теплоты выделится на
резисторе после подключения?
○ 1 Дж
○ 16 Дж
◙ 2 Дж
○ 32 Дж
45. Однозарядные ионы аргона с массовыми
числами 36 и 40 из состояния покоя разгоняются в
электрическом поле разностью потенциалов  = 800 В
и
влетают
в
перпендикулярно
однородное
вектору
магнитное
магнитной
B
поле
индукции,
величина которого B = 0,32 Тл. Описав полукруги, ионы
S
вылетают из области поля двумя пучками. Определить
расстояние S между пучками.
◙ 8,3 мм
○ 2,7 мм
○ 12,4 мм
○ 1,8 мм
46. Плоскость алюминиевого кольца перпендикулярна вектору
индукции однородного магнитного поля. Диаметр кольца d = 25 см,
площадь сечения проволоки S = 3 мм2. Величина магнитной индукции
1.3.4-354
меняется на B = 1,6 Тл за каждую секунду. Определить индукционный
ток в кольце. Удельное электросопротивление алюминия равно 26 нОм.м.
○ 23,0 А
◙ 11,5 А
○ 12,8 А
○ 6,7 А
47. Два точечных заряда имеют массы m = 1 мг и M = 4 мг. Их
заряды q = +1 нКл и Q = -3 нКл соответственно. На каком расстоянии друг
от друга должны быть расположены заряды, чтобы в однородном внешнем
электрическом поле с напряженностью E = 100 кВ/м они имели
одинаковое ускорение. Вектор напряженности электрического поля
параллелен прямой, соединяющей заряды.
◙ 4,4 см
○ 5,3 см
○ 1,6 см
○ 2,8 см
48. Электрон движется в магнитном поле, индукция которого B = 2
мТл, по винтовой линии радиуса R = 2 см с шагом h = 5 см. Определить
скорость электрона.
○ 7,6·103 м/с
◙ 7,6·106 м/с
○ 3,8·106 м/с
○ 3,8·105 м/с
49. Медная трубка, по которой пропускают ток I = 1500 А,
охлаждается водой. Длина трубки L = 35 м, наружный диаметр D = 12 мм,
внутренний d = 10 мм, удельное сопротивление меди равно 17 нОм.м.
1.3.4-355
Температура поступающей в трубку воды равна 120С, выходящей 350С,
удельная теплоемкость воды c = 4200 Дж/кг. Какова скорость
протекающей по трубке воды?
○ 1 м/с
○ 3 м/с
◙ 5 м/с
○ 8 м/с
50. Расстоянием между пластинами плоского конденсатора равно 1
см.
При
облучении
одной
из
пластин
рентгеновскими
лучами
вырывающиеся из нее фотоэлектроны имеют скорость V = 1000 км/с.
Попадая на противоположную пластину электроны ее заряжают. Через
какое время фототок между пластинами прекратится, если ежесекундно с
каждого квадратного сантиметра площади пластины вырывается миллиард
электронов?
○ 96 с
◙ 16 с
○ 8с
○ 2с
51. На горизонтальный вал мотора равномерно наматывается
нитка, на которой подвешен груз массой m = 800 г. Мотор питается
от источника постоянного тока, ЭДС которого равна 12 В, а
внутреннее сопротивление r = 0,4 Ом. Сопротивление цепи мотора
R = 3 Ом. Сколько оборотов в секунду делает якорь мотора, если по
обмоткам мотора течет ток I = 3,3 А? Радиус вала мотора равен 0,5 m
см.
○ 20 с-1
○ 40 с-1
1.3.4-356

○ 5 с-1
◙ 10 с-1
52. Заряженное тело соскальзывает по гладкой
наклонной плоскости параллельной горизонтальному
магнитному полю. Масса тела m = 5 мг, заряд q = 4,9

B
мкКл, угол наклона плоскости к горизонту  = 600,

α
индукция магнитного поля B = 1 Тл. При достижении некоторой скорости
тело отрывается от наклонной плоскости. Найти эту скорость.
◙ 5 м/с
○ 4 м/с
○ 3 м/с
○ 2 м/с
Оптика, фотоны, волны
53. Две плоскопараллельные стеклянные пластины толщиной d1 = 16
мм и d2 = 24 мм сложены вплотную друг к другу. Их показатели
преломления соответственно равны 1,5 и 1,8. На поверхность первой
пластины падает луч света под углом  = 600. Определить смещение луча
относительно
первоначального
направления
после
выхода
с
противоположной стороны из второй пластины в воздух.
○ 11,2 мм
○ 18,3 мм
○ 28,6 мм
◙ 22,4 мм
54. Атомы серебра вылетают из источника в горизонтальном
направлении, обладая кинетической энергией Е = 1,6.10-18 Дж. Определить
1.3.4-357
снижение атомов под действием силы тяжести на расстоянии 5 м от
источника. Молярная масса серебра равна 108 кг/кмоль.
○ 3 мкм
◙ 7 мкм
○ 12 мкм
○ 5 мкм
55. На поверхность воды выливают каплю жидкости массой 1 мг,
которая не смешивается с водой и образует на поверхности пленку,
толщина которой равна одному диаметру молекулы жидкости. Считая, что
молекулы жидкости имеют сферическую форму и их центры образуют на
поверхности квадратную решетку, определить площадь образовавшейся
пленки. Плотность жидкости  = 920 кг/ м3, молярная масса  = 89
кг/кмоль.
◙ 2 м2
○ 12 м2
○ 24 м2
○ 1 м2
56. При напылении серебра на изделие атомы производят давление,
равное 0,1 Па. Плотность серебра  = 10,5 т/м3, молярная масса равна 108
кг/кмоль. Какой толщины получится покрытие за 50 мин, если известно,
что атомы серебра обладают энергией Е = 1,6.10-17 Дж, а их скорость
перпендикулярна поверхности?
○ 1,26 мкм
○ 4,13мкм
◙ 2,13 мкм
○ 6,32 мкм
1.3.4-358
57. Электрический колебательный контур настроен на частоту  = 1
кГц. При колебательном процессе максимальное напряжение на его
конденсаторе равно 100 В. Определить максимальную силу тока к контуре,
если емкость конденсатора С = 2 нФ.
○ 2,52 А
○ 0,63 А
○ 5,04 А
◙ 1,26 А
58. На поверхность металлического электрода в вакуумном
фотоэлементе падает поток света с длиной волны  = 0,4 мкм, мощность
которого Р = 5 мВт. Определить силу фототока насыщения в этом
фотоэлементе, если 5% всех падающих фотонов выбивают из металла
электроны.
◙ 80 мкА
○ 20 мкА
○ 160 мкА
○ 10 мкА
59.
Оптическая
система
состоит
из
собирающей линзы с фокусным расстоянием 30 см
F
и плоского зеркала, находящегося за линзой на
О
расстоянии 15 см от нее. Найти положение
изображения, даваемого этой системой, если
предмет находится перед линзой на расстоянии 15
см от нее.
Зеркало
○ в 40 см перед линзой
◙ в 60 см перед линзой
1.3.4-359
○ в 50 см перед линзой
○ в 70 см перед линзой
60. Каково минимальное расстояние между предметом и его
действительным изображением для линзы с фокусным расстоянием F = 20
см?
○ 40 см
○ 60 см
○ 70 см
◙ 80 см
61. Экран расположен на расстоянии L = 18 см от отверстия, в
которое вставлена рассеивающая линза радиусом r = 3 см. На линзу падает
сходящийся пучок лучей, в результате чего на экране образуется светлое
пятно радиусом R = 5 см. Оказалось, что если линзу убрать, радиус пятна
не изменится. Найти, фокусное расстояние линзы.
○ 10 см
◙ - 15 см
○ -10 см
○ 15 см
62. Экран расположен на расстоянии L = 24 см от отверстия, в
которое вставлена собирающая линза радиусом r = 3 см. На линзу падает
сходящийся пучок лучей, в результате чего на экране образуется светлое
пятно радиусом R = 5 см, Оказалось, что если линзу убрать, радиус пятна
не изменится. Найти, фокусное расстояние линзы.
◙ 20 см
○ -10 см
○ 30 см
1.3.4-360
○ 40 см
63. Точка лежит на оптической оси собирающей линзы, на
расстоянии 40 см от нее. Фокусное расстояние линзы F = 10 см. Точку
перенесли на расстояние 5 см в плоскости, перпендикулярной оптической
оси. На какое расстояние нужно подвинуть линзу в ее плоскости, чтобы
изображение точки получилось в первоначальном месте?
○ 1,75 см
◙ 1,25 см
○ 2,50 см
○ 1,5 см
64. Линза дает действительное изображение предмета с увеличением
k = 3. Как изменится увеличение, если вдвое уменьшить оптическую силу
линзы? Расстояние между предметом и линзой остается неизменным.
Каким станет изображение?
○ мнимое, с увеличением в 1,5 раза
○ действительное, с увеличением в 2 раза
○ мнимое, с увеличением в 4 раза
◙ мнимое, с увеличением в 3 раза
65.
Фотон,
которому
соответствует
длина
волны
10-10 м,
претерпевает упругий центральный удар с первоначально покоившимся
электроном и рассеивается назад. Какую скорость приобретает электрон?
◙ 1,4.10-4 м/с
○ 1,4.10-7 м/с
○ 2,8.10-7 м/с
○ 3,4.10-4 м/с
1.3.4-361
66. Рубиновый лазер дает световой импульс монохроматического
излучения с длиной волны 694 нм. Определить концентрацию фотонов в
луче, если мощность излучения лазера 2 мВт, а площадь сечения луча 4
см2.
○ 5,8.106 м-3
○ 3,4.106 м-3
○ 3,4.1010 м-3
◙ 5,8.1010 м-3
67. Луч лазера с длиной волны излучения, равной 630 нм, имеет вид
конуса с углом при вершине 10-4 рад. Оптическая мощность излучения
лазера равна 3 мВт. На каком максимальном расстоянии наблюдатель
сможет увидеть луч лазера, если глаз воспринимает свет при условии, что
на сетчатку попадает 100 фотонов в секунду? Диаметр зрачка равен 0,5 см.
○ 2500 км
◙ 5000 км
○ 10000 км
○ 7500 км
1.3.4-362
ПРОВЕДЕНИЕ ОЛИМПИАДЫ ПО МАТЕМАТИКЕ И ФИЗИКЕ
Олимпиада по математике
Образец олимпиадного варианта по математике
На
основании
разработанных
методических
материалов
олимпиадных заданий были разработаны варианты отраслевой олимпиады
по математике. Проанализируем прошедшую олимпиаду на основании
следующего
варианта.
Остальные варианты
олимпиадных
заданий
полностью аналогичны. В представленном варианте отражены темы,
рекомендуемые для проведения профильных олимпиад.
Вариант по математике
1. Доказать, что сумма чётного и нечётного чисел нечётна.
2. Линия, заданная на плоскости Oxy уравнением ax  by  c  0,
проходит через I, II и III координатные четверти и не проходит через
начало координат. Найти знак числа ab.
3. В двухэтажном доме на каждом этаже
расположено по 8 комнат так, как показано на
рисунке. В каждой комнате дома висит по 1
лампочке. На втором этаже лампочек горит в 3
раза больше, чем на первом этаже. С левой Кружками
стороны здания горящих лампочек в 3 раза обозначены лампочки,
меньше, чем с правой стороны. С задней стороны какие-то
горящих лампочек в 2 раза больше, чем с передней могут
лампочки
гореть,
а
стороны. Сколько горящих лампочек в доме? какие-то не гореть.
1.3.4-363
Начертить поэтажный план дома с одним из
вариантов горящих лампочек.
4. Решить уравнение
f ( g ( x 1))  3,
где f ( x) 
2x 1
, g ( x) | 3x  2|.
2
5. Около некоторой окружности начерчены 6 окружностей радиуса R
так, что каждая из окружностей радиуса R касается внешним образом
первой окружности и касается двух окружностей радиуса R; и начерчены 4
окружности радиуса r так, что каждая из окружностей радиуса r касается
внутренним образом первой окружности и касается двух окружностей
радиуса r. Найти отношение R/r.
6. При каких значениях параметра a графики
xy 1  0,
x2  y2  8sin2 
разбивают плоскость ровно на 7 частей?
Ответы к олимпиадному варианту по математике
1. Задача на доказательство.
2. Знак минус.
На чертеже показан образец прямой, удовлетворяющей условию
задачи.
1.3.4-364
3. 4.
1 этаж
1
2
4.  ; 
5.
2 этаж
17 .
6
2  1.

5

6.    n;
  n  , n Z .
6
6


1.3.4-365
На чертеже показаны окружности минимального и максимально
радиусов, удовлетворяющих условию задачи.
Анализ решений участников олимпиады, характерные ошибки и
меры по их искоренению
Проанализируем решение каждой задачи и укажем на характерные
ошибки и затруднения школьников при их решении.
Вариант по математике
1. Данная задача на доказательство простого утверждения вызвала
чрезвычайную сложность у многих школьников.
Все, написавшие эту задачу, подтверждали, что сумма чётного и
нечётного чисел нечётна. Однако, лишь у немногих наблюдалось именно
доказательство
этого
утверждения.
Большинство
в
качестве
"доказательства" приводило серию примеров на сложение конкретных
1.3.4-366
двух чисел - чётного и нечётного, откуда и делали вывод, что сумма
чётного и нечётного чисел нечётна.
Некоторые школьники приводили доказательства и в общем виде, но
почему-то в качестве чисел брали два соседних числа.
На примере данной задачи мы видим, что у школьников практически
отсутствует навык доказательства каких-либо утверждений. Фактически
можно сказать, что большинство просто не понимает, что означают слова
"доказать какое-то утверждение".
В школе, начиная с седьмого класса, проходится много теорем, в том
числе и с доказательствами, - от свойств треугольников и теоремы
Пифагора в геометрии до теорем из алгебры и математического анализа.
На разбор этих теорем отведено много времени, однако к окончанию
одиннадцатого класса большинство вообще не понимают смысла слова
"доказать".
Поэтому рекомендуется уделить больше внимания и времени на
разбор доказательств различных математических утверждений. Особенно
на
простых
теоремах,
прививая
школьникам
культуру
строгих
математических рассуждений.
2. Большинство школьников успешно справились с этой задачей,
хотя были и неверные ответы.
Чаще всего решение представляло собой рассмотрение конкретных
уравнений прямых и нахождение требуемого знака на основе этих
примеров. Так как во всех приведенных ими примерах знак требуемого
выражения был одинаковый, на основе этих примеров делался вывод о
знаке исходной задачи.
Лишь немногие приводили решение этой задачи в общем виде, а
вывод о знаке делали на основе точек пересечения прямой с осями
координат.
1.3.4-367
Таким образом, эта задача показала у школьников сложность
рассуждений в общем виде, то есть в буквах, в переменных, а не на
конкретных числах. Многие считали, что если какое-то утверждение верно
для нескольких подобранных ими чисел, то и для любых вообще.
Рекомендуется в школе уделить больше внимания рассуждениям в
общем виде, оперировать с переменными, а не с конкретными числами.
3. Данную задачу решили многие, начертив один из требуемых
вариантов. Однако, собственно решение и обоснование того, что лампочек
именно 4 у многих отсутствовало. Скорее, решение напоминало
графическое угадывание одного из возможных ответов, совершенно не
заботясь о других возможных вариантах.
Тем не менее, часть школьников провела строгие логические
рассуждения и получила, что лампочек может быть именно 4. И уж только
потом привела конкретную реализацию по этажам, что и предполагалось
авторами задачи.
На примере этой задачи видим, что школьникам очень хорошо
удается перебирать случаи, отсеивая случаи, не удовлетворяющие
условиям задачи. Аналитические рассуждения проявили немногие, однако
в силу небольших чисел с помощью перебора всех или части возможных
вариантов большинство школьников справилось с этой задачей.
4. Представленная задача более напоминает обычную школьную
задачу по сравнению со всеми другими. Необычность и олимпиадность
этой задачи связана с операцией взятия функции от функции, что показало
сложности у школьников.
Если же ученик верно записывал само уравнение, то решение для
него уже не представляло никаких сложностей, разве что только обычные
арифметические ошибки.
1.3.4-368
Данную
задачу
решили
менее
половины,
что
показывает
желательность углубленного рассмотрения в школе понятия композиции
функций.
5. Геометрическая задача оказалась сложной для решения. Причем,
сложностью является не идея решения задачи и не вычисления, а всего
лишь чертеж и понимание условия задачи.
Задачу решили единицы, хотя изображение аккуратного чертежа и
применение простейших геометрических свойств позволяет легко решить
задачу.
Представленная задача ярко показала практически полное незнание
школьниками геометрии. Почти все не могли не то что решить задачу, они
не могли даже начертить иллюстрацию, отвечающую условию задачи.
Следовательно, в школьном образовании необходимо более качественно
проходить геометрию, тем более что времени на нее за пять лет изучения
выделяется достаточно.
6. Задача на графическое решение системы уравнений с параметром.
Для ее решения было достаточно начертить входящие в условие гиперболу
и окружность и проследить число областей в зависимости от радиуса
окружности, в который входит параметр.
Оказалось, что многие "не заметили" в этих уравнениях ни
гиперболы, ни окружности. Гипербола была записана в непривычном для
уравнения
гиперболы
виде,
а
радиусом
окружности
являлась
тригонометрическая функция от параметра, что совершенно сбило столку
большинство школьников.
Попытки решить задачу стандартными методами ни к чему не
привели и поэтому многие школьники сочли задачу совершенно
нерешаемой.
Большинство
посчитали
эту
систему
сложной
тригонометрической, даже не подумав, что ее легко можно решить
1.3.4-369
графически, тем более что даже в самом условии сказано про части
плоскости.
Данная задача показала, что школьники с трудом и довольно плохо
освоили понятия функции, графиков, параметров и графических методов
решения задач, что следует учесть при обучении школьников.
Выводы по олимпиаде
Выводы по результатам проведенной олимпиады аналогичны
выводам, изложенным на 3 этапе. Повторим их.
Имеются проблемы, связанные с невнимательностью чтения задач,
особенно текстовых. Потеря в условиях задач важных данных или,
наоборот, прочтение там того, что нет.
Хорошее умение школьниками перебирать случаи, если этих случаев
немного. Но при большом числе вариантов школьники в основном
показали неумение отсеять лишние, заведомо невыполнимые случаи.
Умение решать задачи классическими стандартными методами, если
такие подходят и лежат на поверхности. Если же задача чуть нестандартна,
в ней надо проявить изобретательность, то чаще всего это вызывает
непреодолимые трудности.
Школьники показали плохое знание графиков функций и неумение
решать задачи графическими методами.
Практически полное неумение доказывать и объяснять какие-либо
утверждения, в частности, тяжело даются рассуждения в общем виде и в
задачах с параметрами.
Очень слабое знание геометрии, незнание даже простейших
геометрических понятий и формул.
Сложность с логическими рассуждениями, особенно там, где
присутствует чистая логика, без использования стандартного набора
школьных формул.
1.3.4-370
Итак, на основе всего вышесказанного в школьном образовании
необходимо
уделить
больше
времени
отмеченным
трудностям
и
недостаткам в школьном образовании, более внимательно и усердно
изучать
соответствующие
темы
для
получения
высококлассных
выпускников школ, готовых к поступлению в любые вузы страны.
1.3.4-371
ОЛИМПИАДЫ ПО ФИЗИКЕ
Правовая основа олимпиад по физике
Олимпиады школьников по физике в МГТУ ГА проводились в
соответствии с Положением о Московской региональной олимпиаде
школьников. Согласно Положению, которое действовало до 2008 года,
олимпиада проводилась ежегодно Департаментом образования города
Москвы, Советом ректоров вузов Москвы и Московской области,
окружными управлениями образования, образовательными учреждениями,
при участии образовательных учреждений, научных организаций и
обществ.
проведения
Координацию
Олимпиады
организационно-финансового
осуществлял
по
поручению
обеспечения
Департамента
Московский институт открытого образования (МИОО).
В олимпиаде могли принимать участие слушатели подготовительных
курсов МГТУ ГА и все желающие школьники.
Окружной этап проводился окружным оргкомитетом, по заданиям,
рекомендованным методической комиссией по предмету. По согласованию
с Городским оргкомитетом допускалось проведение окружного этапа
Олимпиады вузом или группой вузов при условии соблюдения Положения,
согласования с Городским оргкомитетом сроков и с методической
комиссией по предмету – заданий Олимпиады.
В соответствии с ранее действовавшим Положением о Всероссийской
олимпиаде школьников городской этап олимпиады города Москвы
приравнивался к 4-му этапу Всероссийской олимпиады школьников.
Окружной этап Олимпиады приравнивался к 3-му этапу Всероссийской
олимпиады школьников. Положениями о Московской региональной
олимпиаде школьников и о Всероссийской олимпиаде школьников для
1.3.4-372
победителей (диплом первой степени) и призёров (дипломы второй и
третьей степени) третьего (окружного) и четвёртого (городского) этапов
олимпиады были предусмотрены льготы при поступлении в вузы.
В 2008/2009 учебном году окружной этап Московской региональной
олимпиады школьников по физике состоялся 2 февраля 2009 года. Для 11классников этот этап проводился на Физическом факультете МГУ им. М.
В. Ломоносова и в вузах города Москвы, в том числе и в МГТУ ГА.
В связи с выходом новых нормативных документов (приказы
Минобрнауки России от 22.10.07 г. № 285 и № 286) порядок проведения
олимпиад в РФ начиная с 2009 года изменился. Московская олимпиада
школьников по физике в настоящее время не имеет статуса этапа
Всероссийской олимпиады школьников. Статус Московской олимпиады
школьников определён приказом Минобрнауки России от 02.09.08 г. №
254, в соответствии с которым ей присвоен третий уровень. При
поступлении в государственные и муниципальные образовательные
учреждения
среднего
профессионального
образования,
а
также
в
государственные и муниципальные образовательные учреждения высшего
профессионального образования победители и призёры олимпиады по
решению образовательного учреждения имеют право в течение одного
года с момента утверждения списков победителей и призёров олимпиады
на получение одной из следующих льгот:
быть приравненными к лицам, набравшим максимальное количество
баллов по единому государственному экзамену по предмету, соответствующему профилю олимпиады;
быть приравненными к лицам, успешно прошедшим дополнительные
вступительные испытания профильной (при поступлении в образовательные учреждения высшего профессионального образования), творческой и (или) профессиональной направленности, предусмотренные
Законом Российской Федерации «Об образовании», по предмету, соот1.3.4-373
ветствующему профилю олимпиады, в порядке, определяемом приёмной
комиссией образовательного учреждения;
быть зачисленными в образовательное учреждение без вступительных
испытаний на направления подготовки (специальности), соответствующие
профилю олимпиады.
Ежегодная отраслевая Олимпиада школьников по физике проводилась
в МГТУ ГА с мая 2009 г. В ней могли принимать участие слушатели
подготовительных курсов МГТУ ГА и все желающие школьники. В
большинстве своем это жители нашего округа Москвы и Подмосковья.
Большой интерес вызывает Олимпиада у школьников Подмосковья,
проживающих в районах аэропортов и авиаремонтных предприятий. В
последние годы стала прослеживаться определенная тенденция – среди
школьников становится престижным попробовать свои силы, блеснуть
смекалкой и получить диплом Олимпиады.
С нормативными документами, определяющими порядок проведения
олимпиады, а также с другой информацией о различных олимпиадах
можно более подробно ознакомиться на портале Российского совета
олимпиад школьников (http://rsr-olymp.ru).
1.3.4-374
ОТРАСЛЕВЫЕ ОЛИМПИАДЫ ШКОЛЬНИКОВ ПО ФИЗИКЕ
В МГТУ ГА
Варианты заданий первой отраслевой
Олимпиады школьниковпо физике в МГТУ ГА
Вариант 1 по физике
1. (10 баллов).
Какова скорость распространения фиолетового света в воде, если
показатель преломления такого света в воде равен 1,343?
2. (20 баллов).
Кусок железа имеет в воде вес Р = 4 Н. Найти его объем. Плотность
воды ρв = 1000 кг/м3, плотность железа ρж = 7800 кг/м3.
3. (20 баллов).
Электрон в однородном магнитном поле с индукцией В = 1 мТл
описал окружность радиусом r = 1 см. Найти скорость электрона.
Элементарный заряд e = 1,6·10-19 Кл, масса электрона me = 9,11·10-31 кг.
4. (25 баллов).
В баллоне находится m = 100 г газа при температуре t = 17 ºС. После
дополнительной подкачки газа в сосуд давление увеличилось на 60%, а
температура повысилась на 30 ºС. Найти массу газа введенного в сосуд при
подкачке.
5. (25 баллов).
Звездная система состоит из двух одинаковых звезд, находящихся на
расстоянии L = 500 млн. км друг от друга. Масса каждой звезды равна
1,5.1034 кг. Найти период обращения звезд вокруг общего центра тяжести.
Гравитационная постоянная G = 6,67·10-11 м3/(кг·с2).
1.3.4-375
Вариант 2 по физике
1. (10 баллов).
Определить энергию фотона, которому соответствует длина волны λ
= 0,6 мкм. Постоянная Планка h = 6,63·10-34 Дж·с, скорость света c = 3·108
м/с.
2. (20 баллов).
На какой глубине в озере давление в 3 раза больше атмосферного.
Атмосферное давление равно Р = 100 кПа, плотность воды ρо = 1000 кг/м3.
3. (20 баллов).
Определить
величину
максимальной
силы,
которую
может
испытывать электрон с кинетической энергией E = 10 эВ при движении в
однородном магнитном поле с индукцией B = 0,2 Тл. Элементарный заряд
e = 1,6·10-19 Кл, масса электрона me = 9,11·10-31 кг
4. (25 баллов).
Свободно падающее тело пролетает некоторый первый участок за
время 2 с, а такой же последний – вдвое быстрее. Найти высоту, с которой
падало тело.
5. (25 баллов).
Сколько молекул воздуха выходит из комнаты объемом V = 120 м3
при повышении температуры от 15 0С до 25 ºС? Число Авогадро NA =
6,02·1023 моль-1, атмосферное давление Р0 = 750 мм.рт.ст., плотность ртути
равна 13600 кг/м3.
Вариант 3 по физике
1. (10 баллов).
Какую работу совершает газ, расширяясь изобарически при давлении
P = 0,2 МПа от объема 1,6 л до 2,5 л?
2. (20 баллов).
1.3.4-376
Абсолютные показатели преломления алмаза и стекла соответственно
равны 2,42 и 1,45. Каково отношение толщин этих веществ, если время
распространения света в них одинаково?
3. (20 баллов).
Однородный стержень массой m = 10 кг
l
и длиной L = 2 м укреплен при помощи
опоры и нити, как показано на рисунке. Определить силу натяжения нити,
если левый конец стержня свисает на l = 40 см.
4. (25 баллов).
Три одинаковых шарика, расположенных вдоль одной прямой,
соединили вместе двумя одинаковыми пружинами жесткостью k = 10 Н/м
каждая. Расстояние между крайними шариками составило 10 см. Затем
всем шарикам сообщили одинаковый заряд, при этом расстояние между
крайними шариками стало равным 11 см. Найти величину заряда,
сообщенному каждому шарику. Электрическая постоянная ε0 = 8,85·10-12
Ф/м.
5. (25 баллов).
Считая, что орбита первого искусственного спутника Земли круговая
радиусом R = 7340 км, определить число оборотов спутника за сутки
вокруг Земли. Принять радиус Земли Rз= 6370 км.
Вариант 4 по физике
1. (10 баллов).
Какова
должна
быть
электроемкость
конденсатора,
который
необходимо включить в колебательный контур, чтобы при индуктивности
катушки L = 0,76 Гн получить колебания частотой  = 400 Гц?
2. (20 баллов).
L
1.3.4-377
К гладкой вертикальной стене на шнуре длиной L = 4 см подвешен
шар массой m = 200 г. Какова сила давления шара на стену, если его
радиус равен 2,5 см? Трением пренебречь.
3. (20 баллов).
Две отрицательно заряженные пылинки находятся на расстоянии L =
2 мм друг от друга и отталкиваются с силой, равной 90 мкН. Считая заряды
пылинок одинаковыми, найти число избыточных электронов на каждой
пылинке. Электрическая постоянная ε0 = 8,85·10-12 Ф/м, элементарный
заряд e = 1,6·10-19 Кл
4. (25 баллов).
Тело свободно падает с высоты h = 100 м. Во сколько раз быстрее оно
проходит последний метр своего пути по сравнению со временем
прохождения первого метра?
5. (25 баллов).
На какую высоту можно было бы поднять груз массой m = 2 т, если
бы удалось полностью использовать энергию, освободившуюся при
остывании стакана воды от температуры t1 = 100 ºС до t2 = 20 ºС? Объем
стакана V = 250 см3, плотность воды равна 1000 кг/м3, ее удельная
теплоемкость св = 4190 Дж/кг.
1.3.4-378
Ответы на задания первой отраслевой
Олимпиады школьниковпо физике в МГТУ ГА
Вариант № 1
1. Ответ: 2,23 108
Вариант № 2
1. Ответ: 3,3 · 10-19
Дж.
2. Ответ: 60 см3
2. Ответ: 20,4 м.
3.Ответ: 1760 км/с.
3. Ответ: 1,9 нН.
4.Ответ: 45 г.
4.Ответ: 30,6 м
5.Ответ:1,6 ·106с.
Вариант № 3
5.Ответ:1026
Вариант № 4
1.Ответ: 180 Дж.
1. Ответ: 0,2 мкФ.
2. Ответ: 1,67 или 0,60.
2. Ответ: 0,82 Н.
3. Ответ: 37,5 Н.
3. Ответ: 1,25.109.
4. Ответ: 0,12 мкКл.
4 .Ответ: в 20 раз.
5. Ответ: ≈ 14.
5.Ответ: 4,3 м.
1.3.4-379
Анализ результатов первой отраслевой
Олимпиады школьников по физике в МГТУ ГА
В первой отраслевой олимпиаде по физике в МГТУ ГА приняли
участие
121
школьник.
В
большинстве
своем
это
слушатели
подготовительных курсов МГТУ ГА, учащиеся школ САО округа Москвы
и Подмосковья. Также большой интерес вызвала Олимпиада у школьников
Подмосковья, проживающих в районах аэропортов и авиаремонтных
предприятий. На олимпиаде было 5 заданий. Вся олимпиадная работа
оценивалась по 100-балльной системе.
Победители и призёры олимпиады определялись в зависимости от
набранных баллов следующим образом:
от 75 до 100 баллов включительно – победитель олимпиады, диплом
1 степени;
от 60 до 74 баллов включительно – призёр олимпиады, диплом 2
степени;
от 50 до 59 баллов включительно – призёр олимпиады, диплом 3
степени.
Трудность отраслевых олимпиадных задач естественным образом
несколько
выше
школьного
и
районного
этапов
Всероссийской
олимпиады. Часть задач отраслевой олимпиады лишь немного сложнее
типовых школьных задач. Решение таких задач, как правило, не должно
представлять трудности для школьника, который успешно освоил
соответствующие разделы школьного курса физики.
В таблице приведены данные статистической обработки результатов
отраслевой олимпиады. На рис. 2 приведена гистограмма распределения
количества набранных балов среди участников.
1.3.4-380
Таблица.
Результаты первой отраслевой
Олимпиады школьников по физике
Процент участников
Набранные 0-10 11-20 21-30 31-40 41-50 51-60 61-70 71-80 81-90 91- 100
баллы
Доля
участников 15,7% 23,0% 8,3% 20,1% 9,1% 14,0% 4,1% 3,3% 1,6% 0,8%
олимпиады
Набранные баллы
Рис.1 Результаты первой отраслевой олимпиады по физике
Анализ этих результатов свидетельствует о том, что в мае месяце на
завершающем этапе довузовской подготовки обнаруживается стабильное
улучшение результативности при решении олимпиадных задач. 24%
участников набрали свыше 50 баллов. Это хороший результат, если учесть,
что задания олимпиады были немного сложнее задний с развернутым
ответом ЕГЭ. Напомним, что на олимпиаде в феврале лишь 16,5%
участников набрали свыше половины баллов от максимального значения.
Налицо
хорошая
корреляция
данных
с
результатами
пробных
тестирований в форме предварительных ЕГЭ, приведенных на рис. 3. От
1.3.4-381
начального уровня, оцениваемого примерно в 50 баллов, учащиеся
стабильно повышают свои результаты.
100
80
Тест 1
60
Тест2
40
Тест 4
Тест3
20
0
уч.1
уч.2
уч.3
Рис. Динамика результатов "пробных" ЕГЭ для трех
школьников
1.3.4-382
Задание второй отраслевой Олимпиады школьников
по физике в МГТУ ГА
(декабрь 2010 г.)
1.
На рисунке 1 показан график
колебаний силы тока в колебательном
контуре с антенной.
электромагнитной
Определите длину
волны,
излучаемой
антенной.
2.
следует
На сколько клеточек и в каком направлении
переместить
глаз
наблюдателя,
чтобы
изображение стрелки в зеркале (рис.2) было видно
глазу полностью?
3.
По
поверхности
стола
движется
с
постоянной скоростью υ черная доска. По доске
движется кусок мела, запущенный по ней так, что в
начальный момент скорость мела относительно
стола u перпендикулярна скорости доски (рис.3). Какой формы след
оставит мел на доске?
4.
Имеются
четыре
тонкие
проволочные
спирали
с
сопротивлениями 10, 20, 30 и 40 Ом. Как из этих спиралей составить
нагревательный элемент с сопротивлением 25 Ом?
5.
Для дальней космической связи используется спутник объемом
V = 1000 м3, наполненный воздухом, находящимся при нормальных
условиях. Оцените время, через которое давление внутри спутника
изменится на 1%, если в корпусе спутника появится отверстие площадью
S =1 см2. Температуру воздуха считать неизменной.
6.
Атмосфера Земли может рассматриваться как гигантская
тепловая машина, в которой роль нагревателя и холодильника играют
экваториальная зона и зоны полюсов, а источником энергии является
1.3.4-383
солнечная радиация. Считая, что полный поток солнечной энергии,
поступающий на Землю, равен Q = 1,7 1017 Вт, а коэффициент полезного
действия
(КПД)
рассматриваемой
«машины»
на порядок
меньше
максимально возможного, оценить среднюю мощность, расходуемую на
образование ветров, в расчете на 1м2
земной поверхности. Обсудить
физические причины, вследствие которых реальный КПД намного меньше
максимально возможного. Считать, что радиус Земли Rз = 6400 км.
Температуру
в
экваториальной
зоне
и
зонах
полюсов
задать
самостоятельно.
1.3.4-384
Решения задач второй отраслевой олимпиады по физике
(декабрь 2010 г.)
Из рисунка 1 определяется
1.
период колебаний  = 4 10 6 с. Так как
длина излучаемой волны  связана со
скоростью
света
и
c
частотой
1
 соотношением c =    , а  = , то  =

c  . Ответ:  =1,2* 103 м.
Ответ:
2.
переместить
в
Глаз
любую
следует
точку
области,
заштрихованной на рисунке 2.
Указание: если учащийся указывает
какую-то
конкретную
заштрихованной
точку
области,
то
из
это
надо
считать правильным ответом и выставлять
за такой ответ наивысший балл.
В системе координат, движущейся с той же скоростью v , что и
3.
доска,
мел
имеет
скорость
u  u  v , где u - скорость мела
сила
трения
направлена
вектору
u ,
мела
о
доску
противоположно
то сила трения
U
U
относительно стола. Так как



Рис. 3
направление скорости мела и,
1.3.4-385
соответственно, направление его движения
изменять не будет. Это
означает, что мел прочертит прямую линию, идущую под углом   arctg
u
v
к вектору - v (рис.3)
4.
Для
того
чтобы
общее
сопротивление
нагревательного
элемента было 25 Ом, нужно составить схему, состоящую из двух
параллельно соединенных участков. Первый участок должен состоять из
последовательно соединенных спиралей с сопротивлениями 10 и 40 Ом, а
второй – из последовательно соединенных спиралей с сопротивлениями
20 и 30 Ом.
5.
Число молекул, которые за время  проходят через отверстие
площадью S , перпендикулярное оси x , направленной перпендикулярно
n
2
плоскости отверстия, равно Z  Vx S , где n - концентрация молекул, Vx среднее значение модуля проекции скорости молекул на ось x .. При этом
число молекул в единице объема изменяется на n 
 2
Z n SVx

, Откуда
V 2 V
n V
.
n SVx
По условию задачи температура воздуха в спутнике остается
неизменной. Как следует из уравнения Менделеева-Клапейрона, в этом
случае давление пропорционально плотности газа и, соответственно,
концентрации молекул. Поэтому
p n

 0, 01 .
p
n
Для оценки можно считать, что 3 Vx 2  U 2 , где U - модуль средней
скорости движения молекул, поэтому Vx 
масса воздуха. Следовательно,   2
pV
pS

RT
1
U
3
RT

где  - молярная
 70 0с.
1.3.4-386
6. Принимая ориентировочно (школьники, естественно могут
выбрать несколько иные цифры) температуру на экваторе эк около 40
градусов Цельсия (примерно 313К), а для разности температур между
тропической и полярной областями   50K , получаем по теореме Карно
max =

 0,16.
 эк
Мощность N , преобразуемая ежесекундно в энергию
ветров, определяется из соотношения N =


Q =0,1 max 2 Q  5 106 Вт / км 2
2
4 Rз
4 Rз
.
В качестве причин различия  и max можно рассмотреть поглощение
солнечной энергии в областях высоких широт, избыточное тепловое
излучение (то есть превышающее среднее) из области низких широт,
теплопроводность,
неполное
поглощение
солнечного
излучения
в
атмосфере и др.
1.3.4-387
Анализ результатов второй отраслевой
Олимпиады школьников по физике в МГТУ ГА
(декабрь 2010 г.)
Во второй отраслевой олимпиаде по физике в МГТУ ГА приняли
участие 69 школьник. Как обычно в большинстве своем это слушатели
подготовительных курсов МГТУ ГА, учащиеся школ САО округа Москвы
и Подмосковья. Также большой интерес вызвала Олимпиада у школьников
Подмосковья, проживающих в районах аэропортов и авиаремонтных
предприятий. На олимпиаде было 6 заданий. Вся олимпиадная работа
оценивалась по 30-балльной системе.
Победители и призёры олимпиады определялись в зависимости от
набранных баллов следующим образом:
от 23 до 30 баллов включительно – победитель олимпиады, диплом 1
степени;
от 19 до 22 баллов включительно – призёр олимпиады, диплом 2
степени;
от 15 до 18 баллов включительно – призёр олимпиады, диплом 3
степени.
В таблице приведены данные статистической обработки результатов
второй
отраслевой
олимпиады.
На
рис.
приведена
гистограмма
распределения количества набранных балов среди участников.
Таблица.
Результаты второй отраслевой Олимпиады школьников по физике
(декабрь 2010 г.)
Набранные
0-2
3-6
7-10 11-14 15-18 19-22 23-26 27-30
баллы
Число
участников
олимпиады
3
8
18
17
14
5
4
0
1.3.4-388
Число участников
набравших баллы
20
15
10
18
5
0
17
14
8
5
3
0-2
4
0
3-6
7-10 11-14 15-18 19-22 23-27 28-30
Набранные баллы
Рис. 3. Результаты второй отраслевой олимпиады по физике (2010 г.)
Анализ приведенных результатов второй отраслевой олимпиады по
физике свидетельствует о том, что в декабре месяце знания и умения
школьников соответствуют начальному этапу подготовки к поступлению в
вузы. Распределение участников по набранным баллам соответствует
нормальному закону Гаусса. Участники слабо мотивированы. Среди
участников
олимпиады
преобладают
школьники
со
средними
способностями. Большинство из них справились только с заданиями
характерными для уровня заданий ЕГЭ. Налицо хорошая корреляция
данных с результатами пробных тестирований в форме предварительных
ЕГЭ. Одаренных или талантливых участников значительно меньше, что и
соответствует нормальному распределению людей по способностям.
В марте 2011 г. состоится второй тур олимпиады. К нему
допускаются победители и призеры первого тура.
1.3.4-389
Выводы
Рассмотренные подходы позволяют при их систематическом и
целенаправленном осуществлении на новой основе конструировать
научно-методическую
базу
проведения
профильных
Олимпиад
по
математике и физике в соответствии с основными целями и принципами
государственной научно-технической политики с учетом действующей
школьной программы. Это сказывается на результативности олимпиадного
движения:
знания
обобщенными,
приобретают
комплексными,
системность,
усиливаются
умения
становятся
мировоззренческая
направленность познавательных процессов учащихся, более эффективно
формируются их убеждения, достигается всестороннее развитие личности.
1.3.4-390
75.1.3.5
ЛИТЕРАТУРА
[1] Рябчун И.П., Моисеев С.К., Мальчинский Ф.В., Скалецкий В.В.,
Пивень В.А. Особенности преподавания физики в летных учебных
заведениях. Труды Краснодарского ВАИ. Вып. V, Краснодар 2002.
[2] Смирнов С.Д. Педагогика и психология высшего образования. От
деятельности к личности. М.: «Academa» 2001. С.59-71
[3] Анташева Н.А. Концептуальные основы педагогической
аксиологии. Педагогика. №8, 2002. С. 8-13.
[4] Закон Российской федерации «Об образовании». Высшее
образование в России. №3, 1992. С. 5-36.
[5] Физика в Политехническом. Из века 20-го – в век 21-й. СПБ.: Издво СПбГПУ, 2002. С.34.
[6] Шаскольская М. П., Эльцин И. А. Сборник избранных задач по
физике / Под ред. С. Э. Хайкина. – М.-Л.: Гостехиздат, 1949. –132 c. (и все
последующие издания до 5-го, переработанного, М.: Наука, 1986).
[7] Зубов В. Г., Шальнов В. П. Задачи по физике. – М.: Гостехиздат,
1952. – 320 с. (и все последующие издания до 11-го, М.: Новая волна,
2000).
[8] Бендриков Г. А., Буховцев Б. Б., Керженцев В. В., Мякишев Г. Я.
Задачи по физике для поступающих в вузы. – М.: Наука, 1980. - 384 с. (и
все последующие издания до 10-го, М.: Физматлит, 2003).
[9] Буховцев Б. Б., Кривченков В. Д., Мякишев Г. Я., Сараева И. М.
Сборник задач по элементарной физике: Пособие для самообразования. –
М.: Наука, 1964. - 440 с. (и все последующие издания до 7-го, М.: УНЦ ДО
МГУ, 2004).
[10] Буздин А. И., Ильин В. А., Кривченков И. В., Кротов С. С, Свешников Н. А. Задачи московских физических олимпиад / Под ред. С. С.
Кротова. – М.: Наука. Гл. ред. физ.-мат. лит., 1988. - 192 с. -(Библиотечка
«Квант». Вып. 60.)
[11] Варламов С. Д., Зинковский В. И., Семенов М. В., Старокуров Ю.
В., Шведов О. Ю., Якута А. А. Задачи Московских городских олимпиад по
физике. 1986 - 2005. / Под ред. М. В. Семенова, А. А. Якуты. - М.: Изд-во
МЦНМО, 2006. - 616 с.
[12] Задачи Московской региональной олимпиады школьников по
физике 2006 года. / Под ред. М. В. Семенова, А. А. Якуты. - М.: Изд-во
МЦНМО, 2007. - 56 с.
[13] Буздин А. И., Зильберман А. Р., Кротов С. С. Раз задача, два задача... - М.: Наука. Гл. ред. физ.-мат. лит., 1990. - 240 с. - (Библиотечка
«Квант». Вып. 81.)
1.3.4-391
[14] Слободецкий И. Ш., Асламазов Л. Г. Задачи по физике. - М.: Наука. Гл. ред. физ.-мат. лит., 1980. - 176 с. - (Библиотечка «Квант». Вып. 5). А
также 2-е изд. - М.: Бюро Квантум, 2001. - 160 с. (Библиотечка «Квант».
Вып. 86).
[15] Балаш В. А. Задачи по физике и методы их решения. - М.:
Просвещение, 1964 (и все последующие издания до 4-го, М.:,
Просвещение, 1983).
[16] Задачи по физике: Учебное пособие / Под ред. О. Я. Савченко. –
4-е изд., испр. - СПб.: Лань, 2001. - 368 с.
[17] Слободецкий И. Ш., Орлов В. А. Всесоюзные олимпиады по
физике: Пособие для учащихся 8-10 кл. сред. школы. - М.: Просвещение,
1982. - 256 с.
[18] Всероссийские олимпиады по физике. 1992-2004 / Под ред. С. М.
Козела, В. П. Слободянина. - 2-е изд., доп. - М.: Вербум-М, 2005. - 534 с.
[19] Кабардин О. Ф., Орлов В. А. Международные физические
олимпиады школьников / Под. ред. В. Г. Разумовского. - М.: Наука. Гл.
ред. физ.-мат. лит., 1985. - 160 с. - (Б-чка «Квант». Вып. 43)
[20] Физика. 10-11 кл.: Сборник задач и заданий с ответами и
решениями. Пособие для учащихся общеобразоват. учреждений / С. М.
Козел, В. А. Коровин, В. А. Орлов. - М.: Мнемозина, 2001. -254 с.
[21] Лукашик В. И. Физическая олимпиада в 6-7 классах средней школы: Пособие для учащихся. - 2-е изд., перераб. и доп. - М.: Просвещение,
1987. - 192 с.
[22] Григорьев Ю. М., Муравьев В. М., Потапов В. Ф. Физика. Олимпиадные задачи по физике. Международная олимпиада «Туймаада». - М.:
МЦНМО, 2006. - 160 с.
[23] Гольдфарб Н. И. Физика. Задачник. 10-11 кл.: пособие для общеобразовательных учреждений. - М.: Дрофа, 2006. - 398 с. (и все
предыдущие издания).
[24] Страница Московской физической олимпиады на сервере
Кафедры
общей
физики
Физического
факультета
МГУ:
http://genphys.phys.msu.ru/ol/
[25]
Веб-сайт
«Олимпиады
для
школьников»:
www.mccme.ru/olimpiads/
[26] Материалы журнала «Квант» в интернете: www.kwant.mccme.ru/
[27] Архив материалов газеты «Физика» (Издательский дом «Первое
сентября»): http:// archive.1september.ru/fiz/
[28] Интернет-библиотека МЦНМО: www.ilib.mccme.ru/
[29] IPhO - International Physics Olympiads. Материалы международных
физических
олимпиад
(на
английском
языке).
www.jyu.fi./tdk/kastdk/olimpiads/
1.3.4-392
[30] Отличник ЕГЭ. Физика. Решение сложных задач. Под ред.
В.А.Макарова, М.В. Семенова, А.А. Якуты; ФИПИ. –М.: Интеллект–центр,
2010.–368 с.
[31] Задачи олимпиады Кенгуру. Решения и ответы. Интернет-сайт:
http://intelmath.narod.ru/kangaroo-problems.html.
[32] Задачи математических олимпиад школьников. Интернет-сайт:
http://e-ypok.ru/book/export/html/287
[33] Задачи олимпиад. Интернет-сайт: http://www.problems.ru
1.3.4-393
Download